TARGET PRELIMS 2017neoias-838f.kxcdn.com/images/pdf/prelims-2017/Important...Colour-One...

39
NEO IAS 0484-3190310, 9446331522, 9446334122 9446331522 Page 1 www.neoias.com | www.youtube.com | www.facebook.com/neoias | www.twitter.com/neoias 100 IMPORTANT ENVIRONMENTAL SCIENCE QUESTIONS FROM NEO IAS MODEL TESTS 1. Consider the following statements with regard to recently passed Compensatory Afforestation Fund (CAF) Bill, 2016: 1. It proposes to set up the Compensatory Afforestation Fund Management and Planning Authority. 2. Compensatory Afforestation Fund (CAF) is derived from the charge levied for destroying forest based on their Net Present Value. 3. The bill proposes to use the fund for the afforestation purpose only. 4. It proposes to establish the National CAF under the public account of India and State CAF under the public account of each state. Which of the statements given above is/are correct? (a) 1 and 2 only (b) 3 and 4 only (c) 1, 2 and 4 only (d) 1, 2, 3 and 4 Answer (c) Explanation 3 rd statement is wrong. Although the Funds will be primarily spent on afforestation to compensate for loss of forest cover, regeneration of forest ecosystems, the fund is also used for the wildlife protection and infrastructure development 2. Consider the following with reference to Gangetic Dolphin, the National Aquatic animal of India: 1. The developmental project in Ganga River for shipping, especially due to the National Waterway I Project. 2. Depletion of prey base. 3. Accidental mortality in fishing nets. 4. Accidents with vessel propellers. Which of the above are likely causes for reduction in numbers of the animal? (a) 1 and 2 only (b) 2 only (c) 1 and 3 only (d) 1, 2, 3 and 4 Answer (d) Explanation The development of the Ganga for shipping is seen by wildlife conservationists as the single-largest threat to the survival of Gangetic Dolphins, whose numbers are declining in most parts of their natural habitat. This is mainly due to construction of dams and barrages on the river. Aside from losing habitat to increased developmental work on the river, the dolphins also suffered due to depletion of prey base, accidental mortality in fishing nets and accidents with vessel propellers. The large-scale modification of the river, the proposed movement of numerous ships, may well sound the death knell of the species. The species are practically blind, and rely on bio-sonar method to move around. The ships‘ noise-levels would disrupt the ability to navigate, and find prey. 3. With reference to Particulate Matter(PM2.5) Consider the following statements: 1. PM is the mixture of solid particles and liquid droplets found in the air. 2. The PM2.5 causes chronic illnesses such as lung cancer, chronic obstructive pulmonary disease (COPD) and cardiovascular diseases, that cause premature death. 3. A lot of the fine particulate matter comes from fuel combustion, both from mobile sources such as vehicles and from stationary sources such as power plants, industry, households or biomass burning. Which of the statements given above is/are correct? (a) 1 only (b) 1 and 3 only (c) 2 and 3 only (d) 1, 2 and 3 Answer: (d) Explanation PM stands for particulate matter (also called particle pollution): the term for a mixture of solid particles and liquid droplets found in the air. Some particles, such as dust, dirt, soot, or smoke, are large or dark enough to be seen with the naked eye. Others are so small they can only be detected using an electron microscope. The impact of fine particulate matter (PM2.5) highlighted by the World Health Organisation (WHO) study is felt through a broad spectrum of acute and chronic illnesses that cause TARGET PRELIMS 2017 0484-3190310 9446331522 9446331522

Transcript of TARGET PRELIMS 2017neoias-838f.kxcdn.com/images/pdf/prelims-2017/Important...Colour-One...

NEO IAS 0484-3190310, 9446331522, 9446334122 9446331522 Page 1 www.neoias.com | www.youtube.com | www.facebook.com/neoias | www.twitter.com/neoias

100 IMPORTANT ENVIRONMENTAL SCIENCE QUESTIONS FROM NEO IAS MODEL TESTS

1. Consider the following statements with regard to recently passed Compensatory Afforestation Fund (CAF) Bill, 2016:

1. It proposes to set up the Compensatory Afforestation Fund Management and Planning Authority.

2. Compensatory Afforestation Fund (CAF) is derived from the charge levied for destroying forest based

on their Net Present Value.

3. The bill proposes to use the fund for the afforestation purpose only.

4. It proposes to establish the National CAF under the public account of India and State CAF under the public account of each state.

Which of the statements given above is/are correct?

(a) 1 and 2 only (b) 3 and 4 only

(c) 1, 2 and 4 only (d) 1, 2, 3 and 4

Answer (c)

Explanation

3rd statement is wrong. Although the Funds will be primarily spent on afforestation to compensate for

loss of forest cover, regeneration of forest ecosystems, the fund is also used for the wildlife protection

and infrastructure development

2. Consider the following with reference to Gangetic Dolphin, the National Aquatic animal of India:

1. The developmental project in Ganga River for shipping, especially due to the National Waterway I

Project.

2. Depletion of prey base. 3. Accidental mortality in fishing nets.

4. Accidents with vessel propellers.

Which of the above are likely causes for reduction in numbers of the animal?

(a) 1 and 2 only (b) 2 only

(c) 1 and 3 only (d) 1, 2, 3 and 4

Answer (d)

Explanation

The development of the Ganga for shipping is seen by wildlife conservationists as the single-largest

threat to the survival of Gangetic Dolphins, whose numbers are declining in most parts of their natural habitat. This is mainly due to construction of dams and barrages on the river. Aside from losing

habitat to increased developmental work on the river, the dolphins also suffered due to depletion of

prey base, accidental mortality in fishing nets and accidents with vessel propellers. The large-scale

modification of the river, the proposed movement of numerous ships, may well sound the death knell

of the species. The species are practically blind, and rely on bio-sonar method to move around. The ships‘ noise-levels would disrupt the ability to navigate, and find prey.

3. With reference to Particulate Matter(PM2.5) Consider the following statements:

1. PM is the mixture of solid particles and liquid droplets found in the air.

2. The PM2.5 causes chronic illnesses such as lung cancer, chronic obstructive pulmonary disease (COPD) and cardiovascular diseases, that cause premature death.

3. A lot of the fine particulate matter comes from fuel combustion, both from mobile sources such as

vehicles and from stationary sources such as power plants, industry, households or biomass

burning.

Which of the statements given above is/are correct?

(a) 1 only (b) 1 and 3 only (c) 2 and 3 only (d) 1, 2 and 3

Answer: (d)

Explanation PM stands for particulate matter (also called particle pollution): the term for a mixture of solid particles

and liquid droplets found in the air. Some particles, such as dust, dirt, soot, or smoke, are large or

dark enough to be seen with the naked eye. Others are so small they can only be detected using an

electron microscope. The impact of fine particulate matter (PM2.5) highlighted by the World Health

Organisation (WHO) study is felt through a broad spectrum of acute and chronic illnesses that cause

TARGET PRELIMS 2017

0484-3190310

9446331522

9446331522

NEO IAS 0484-3190310, 9446331522, 9446334122 9446331522 Page 2 www.neoias.com | www.youtube.com | www.facebook.com/neoias | www.twitter.com/neoias

premature death. These include lung cancer, chronic obstructive pulmonary disease (COPD) and cardiovascular diseases. Of all of pollutants, fine particulate matter has the greatest impact on health.

A lot of the fine particulate matter comes from fuel combustion, both from mobile sources such as

vehicles and from stationary sources such as power plants, industry, households or biomass burning.

4. With reference to Laggar falcon, an indigenous species in India, consider the following statements: 1. Laggar Falcon is a raptor species with white and grey plumage, which can hunt and fly at speeds of

up to 180 kmph

2. The species usually nest on rock cliffs unreachable to humans

3. They have fallen victim to granite sand quarrying

Select the correct answer using the code given below. (a) 1 only (b) 1 and 2 only

(c) 2 and 3 only (d) 1, 2 and 3

Answer: (d)

Explanation Laggar Falcons are an indigenous raptor species with white and grey plumage, which can hunt and fly

at speeds of up to 180 kmph. Indiscriminate sand quarrying on the Vaigai river bed, and granite

quarrying in many parts of Madurai, have spelt disaster for many raptor species and today, only two

Laggar Falcons survive on the rock cliffs of Arittapatti near Melur in the district. The species nest only

on rock cliffs unreachable to humans. Due to sand quarrying, Madurai lost its Vaigai-based

biodiversity, including hundreds of Brahminy Kites, Black Kites and native fishes. Stone quarrying in Keezhavazhavu and Melavazhavu destroyed the habitat and nesting spots of raptors and other birds,

including eagles, falcons and owls. The use of explosives for granite quarrying contributed to the

decline of the birds. Felling of palm trees, which is a nesting spot of many raptor species, is also one of

the reasons for the disappearance of the birds.

5. With reference to Pika, a species which was discovered recently, consider the following statements:

1. It is a mammal belonging to the rabbit and hare family.

2. Pikas are Pioneer species and ecosystem engineers in an ecosystem

3. It is discovered in the Himalayas in Sikkim.

Which of the statements given above is/are correct? (a) 1 only (b) 2 and 3 only

(c) 1 and 3 only (d) 1, 2 and 3

Answer: (c)

Explanation Scientists claim to have discovered a new species of Pika, a mammal belonging to the rabbit and hare

family (Lagomorpha), in the Himalayas in Sikkim. After six years of research, a team of international

collaborators, led by scientists from Bengaluru‘s National Centre for Biological Sciences (NCBS)

announced the discovery. Pikas are members of the rabbit family and live in the mountains or in

temperate regions. Pikas are a keystone species and ecosystem engineers. Pikas do not hibernate unlike other mammalian species inhabiting such cold climates.

6. In the cities of our country, which among the following atmospheric gases are normally considered in

calculating the value of Air Quality Index?

1. Ozone 2. Carbon dioxide

3. Particulate matter 10 (PM10)

4. Ammonia

5. Lead

Select the correct answer using the code given below. (a) 1, 2 and 3 only (b) 2, 4 and 5 only

(c) 1, 3, 4 and 5 only (d) 1, 2, 3, 4 and 5

Answer: (c)

Explanation

The new National Air Quality Index (AQI) launched in 2014 can be described as a ‗One Number- One

Colour-One Description‘ for the common man to judge the air quality within his vicinity. Air Quality Index (AQI) is a tool for effective dissemination of air quality information to people. The formulation of

the index was a continuation of the initiatives under Swachh Bharat Mission. The current

measurement index for air quality will consider eight pollutants or parameters (PM10, PM2.5, Nitrogen

dioxide (NO2), Sulfur dioxide (SO2), Carbon monoxide (CO), ozone (O3), Ammonia (NH3), and Lead

(Pb)) for which short-term (up to 24-hourly averaging period) National Ambient Air Quality Standards are prescribed. There are six AQI categories, namely Good, Satisfactory, Moderately polluted, Poor,

Very Poor, and Severe.

NEO IAS 0484-3190310, 9446331522, 9446334122 9446331522 Page 3 www.neoias.com | www.youtube.com | www.facebook.com/neoias | www.twitter.com/neoias

7. With reference to the Convention on International Trade in Endangered Species of Wild Fauna and

Flora (CITES), consider the following statements: 1. It is a non – governmental organisation which aims to ensure that international trade of wildlife

does not threaten their survival.

2. The species covered by CITES are listed in three Appendices, according to the degree of protection

they need.

3. CITES has its own enforcement authority to regulate international trade in wild fauna and flora.

4. CITES regulates international trade in wild fauna and flora listed in its Appendices on the basis of a system of permits and certificates.

Which of the above statements given above is/are correct?

(a) 1 only (b) 2 and 4 only

(c) 2 and 3 only (d) 1, 2, 3 and 4

Answer: (b)

Explanation

CITES is an international agreement between governments which aims to ensure that international

trade in specimens of wild animals and plants does not threaten their survival. The purpose of CITES

is to ensure that wild fauna and flora in international trade are not exploited unsustainably. The species covered by CITES are listed in three Appendices, according to the degree of protection they

need. CITES regulates international trade in wild fauna and flora listed in its Appendices on the basis

of a system of permits and certificates which are issued when certain conditions are met, and which

must be presented when leaving and entering a country. CITES is not having any enforcement

authority that means they do not have its own police force, they depend on laws and enforcement procedures of each nation. The 17th meeting of the Conference of the Parties to CITES (CoP17) took

place in Johannesburg, South Africa from 24 September to 5 October 2016.

8. Consider the following States:

1. Assam

2. West Bengal

3. Himachal Pradesh In which of the above States does tropical evergreen vegetation occur?

(a) 1 only (b) 2 and 3 only

(c) 1 and 2 only (d) 1, 2 and 3

Answer: (c) Explanation

These forests are found in the western slope of the Western Ghats, hills of the north-eastern region

and the Andaman and Nicobar Islands. Forest found in heavy rainfall areas (more than 200cm) with

short dry seasons. It has luxuriant vegetation from trees to creepers. Ebony, mahogany, rosewood,

rubber and cinchona are some of the commercially important trees found in these forests. Elephants, monkey, lemur and deer make these forests their habitat. The jungles of Assam and West Bengal

house the one horned rhinoceros. Plenty of birds, bats, sloth, scorpions and snails are also found in

these rain forests.

NEO IAS 0484-3190310, 9446331522, 9446334122 9446331522 Page 4 www.neoias.com | www.youtube.com | www.facebook.com/neoias | www.twitter.com/neoias

9. With reference to Phytoplankton consider the following statements:

1. It has a great role in making the ocean the largest carbon sink.

2. They are the first trophic level component of aquatic food chain.

3. It uses bioluminescence phenomenon for communication when they have stresses in the water

body.

Which of the statements given above is/are correct? (a) 1 only (b) 1 and 2 only

(c) 2 and 3 only (d) 1, 2 and 3

Answer: (d) Explanation

Phytoplankton is the plant like organisms of the water that carry out photosynthesis and float in the

upper areas of the world‘s ocean. They are the producers present in the aquatic ecosystem.

Phytoplankton, also known as microalgae, are similar to terrestrial plants in that they contain

chlorophyll to capture sunlight, and they use photosynthesis to turn it into chemical energy in order to

live and grow. Phytoplankton is responsible for most of the transfer of carbon in the carbon dioxide from the atmosphere to the ocean. Worldwide, this ―biological carbon pump‖ transfers about 10

gigatonnes of carbon from the atmosphere to the deep ocean each year. Even small changes in the

growth of phytoplankton may affect atmospheric carbon dioxide concentrations, which would feed

back to global surface temperatures. Certain creatures both on land and sea can produce light

through chemical reactions taking place within their bodies known as Bioluminescence. Bioluminescent phytoplankton occur in all the world‘s oceans. The most common of these are

Dinoflagellates which are tiny unicellular marine plankton also known as fire plants. Bioluminescence

is used to evade predators and acts as a defense mechanism in dinoflagellates. Dinoflagelletes produce

light when disturbed and will give a light flash lasting a fraction of a second. The flash is meant to

attract a predator to the creature disturbing or trying to consume the dinoflagellate. The light flash

also surprises the predator causing it to worry about other predators attacking it, making the predator less likely to prey on the dinoflagellate.

10. Consider the following statements regarding ecological succession:

1. Ecological succession is the gradual and progressive replacement of one community by another.

2. As ecological succession proceeds the number and types of animals also change.

3. The xerarch succession starts from a wetter area and ends in a mesic state.

Which of the statements given above is/are correct?

(a) 1 only (b) 1 and 2 only

(c) 2 and 3 only (d) 1, 2 and 3

Answer: (b) Explanation

The gradual and progressive replacement of one community by another, till the development of a stable

community in that area, or a process through which ecosystems tend to change over a period of time is

called as ecological succession. Succession can be related to seasonal environmental changes, which

create changes in the community of plants and animals living in the ecosystem. Hydrarch Succession takes place in wetter areas whereas Xerarch succession takes place in dry areas. Pioneer species in

Hydrarch succession are usually the small phytoplankton and that in Xerarch succession are usually

lichens.

11. Consider the following statements regarding Ecological Niche: 1. Ecological niche is the physical position and functional role of a species within the ecosystem.

2. No two species in a habitat can have the same niche.

3. An ecological niche describes how an organism or population responds to the distribution of

resources and competitors.

Which of the statements given above is/are correct? (a) 1 and 2 only (b) 2 only

(c) 3 only (d) 1, 2 and 3

Answer: (d)

Explanation An ecological niche is the role and position a species has in its environment (ecosystem); how it meets

its needs for food and shelter, how it survives, and how it reproduces. Ecological niche is the physical

space or position and functional role of a species within the community or ecosystem. A species niche

includes all of its interactions with the biotic and abiotic factors of its environment. A niche is unique

for a species while many species share the habitat. No two species in a habitat can have the same niche. This is because of the competition with one another until one is displaced. The description of a

niche may include descriptions of the organism's life history, habitat, and place in the food chain.

NEO IAS 0484-3190310, 9446331522, 9446334122 9446331522 Page 5 www.neoias.com | www.youtube.com | www.facebook.com/neoias | www.twitter.com/neoias

More formally, the niche includes how a population responds to the abundance of its resources and enemies. The abiotic or physical environment is also part of the niche because it influences how

populations affect, and are affected by, resources and enemies.

12. Which one of the following is not a correct statement about coastal wetlands?

(a) Coastal habitats like mangroves and salt marshes play an important role in reducing climate

change

(b) Mangroves and coastal wetlands store three to five times more carbon per equivalent area than

tropical forests

(c) Carbon sequestration is the process of releasing carbon dioxide to the atmosphere

(d) Coastal blue carbon is the carbon captured by living coastal and marine organisms and stored in

coastal ecosystems

Answer: (c) Explanation

Healthy coastal habitat is not only important for seafood and

recreation, it also plays an important role in reducing

climate change. Salt marshes, mangroves, and seagrass

beds absorb large quantities of the greenhouse gas carbon dioxide from the atmosphere and store it, thus decreasing

the effects of global warming. These types of habitat are

known as carbon sinks and contain large stores of carbon

accumulated over hundreds to thousands of years. Using

more scientific lingo, coastal blue carbon is the carbon

captured by living coastal and marine organisms and stored in coastal ecosystems. Salt marshes, mangroves, and

seagrass beds play two important roles:

Carbon sequestration—the process of capturing carbon

dioxide from the atmosphere, measured as a rate of carbon uptake per year

Carbon storage—the long-term confinement of carbon in plant materials or sediment, measured as a total weight of carbon stored

Current studies suggest that mangroves and coastal wetlands annually sequester carbon at a rate ten

times greater than mature tropical forests. They also store three to five times more carbon per

equivalent area than tropical forests. Most coastal blue carbon is stored in the soil, not in above-

ground plant materials as with tropical forests. Coastal habitats are important for capturing carbon—but their destruction poses a great risk. When

these habitats are damaged or destroyed, it is not only their carbon sequestration capacity that is lost.

Carbon stored in the habitats is also released, increasing levels of greenhouse gases in the

atmosphere. Unfortunately, coastal habitats around the world are being lost at a rapid rate, largely

due to coastal development for housing, ports, and commercial facilities.

13. With reference to wetlands, consider the following statements:

1. Wetlands, especially peatlands, are the biggest store of carbon on land.

2. The draining and degradation of wetlands will turn them into a net source of greenhouse gas

emissions.

3. Destruction of wetland could lead to flooding.

Which of the above statements given above is/are correct?

(a) 1 only (b) 2 and 3 only (c) 1 and 2 only (d) 1, 2 and 3

Answer: (d)

Explanation

Wetlands, especially peatlands, are the biggest store of carbon on land. The draining and degradation of wetlands turns them into a net source of greenhouse gas emissions. The restoration of damaged

wetlands can halt emissions of carbon dioxide and even reverse them, causing carbon removal from

the atmosphere. Emissions of nitrous oxide and methane can also be reduced or halted by restoration.

NEO IAS 0484-3190310, 9446331522, 9446334122 9446331522 Page 6 www.neoias.com | www.youtube.com | www.facebook.com/neoias | www.twitter.com/neoias

14. With reference to productivity, consider the following statements:

1. Primary productivity is the amount of biomass or organic matter produced per unit area over a

period of time in the first trophic level.

2. Photosynthetic capacity of plants and photosynthetic active radiation are the only factors which

determine the primary productivity.

Which of the statements given above is/are correct?

(a) 1 only (b) 2 only

(c) Both 1 and 2 (d) Neither 1 nor 2

Answer: (a)

Explanation

Primary productivity means the rate of food produced by producers (autotrophs) or the rate of solar energy trapped at first trophic level. Primary productivity is again classified in to two, Gross primary

productivity and Net primary productivity. Gross Primary Productivity (GPP) means the total amount of

energy produced by producers (autotrophs) at trophic level one. Or in another words it is the rate at

which an ecosystem‘s producers convert solar energy into chemical energy as biomass. This gross

productivity depends up on photosynthetic capacity of producers and environmental factors including

climatic conditions such as temperature, rainfall and total solar radiation and other nutrient materials of the abiotic environment such as nitrogen, phosphorus and sulphur.

15. Which of the following is/are unique characteristic/characteristics of deciduous forests?

1. They have nutrient rich soil due to less leaching.

2. Since it is getting a high amount of rainfall annually they are deficient in nutrients.

3. Presence of tall, closely set trees with crowns forming a continuous canopy.

4. Presence of largest number of domesticated animals.

Select the correct answer using the codes given below.

(a) 1 only (b) 2 and 3 only

(c) 1 and 4 only (d) 1, 2, 3 and 4

Answer: (c)

Explanation

Trees in deciduous forest are normally deciduous, because of the dry period summer, during which

they shed their leaves to withstand the drought. Normally, it extends between 100 and 250 N-S

latitude. They are found in South East Asia, Central and South America, South Eastern USA, South-Eastern and Western Africa, Northern Australia, Southern Brazil, Burma, Taiwan and southern china.

These are the most widespread forest in India. They are rich in nutrients than equatorial rain forest.

The number of plants species is less in the tropical deciduous forest than the tropical evergreen

rainforest. So the number of animal species is less than that of the evergreen rain forest. Since the

density of plants is lower than the rainforest, there is less competition among plants for getting

sunlight, so the trees are of medium height (25-45m). They are occurring in regions where total annual rainfall is about 100cm. This biome has the largest number of domesticated animals, because of the

development of agriculture.

16. What is/are unique about Khur, a breed found in India?

1. It is a mammal which can race at nearly 60 kmph.

2. It is native to the Little Rann of Kutch.

3. The species has come out from the brink of extinction, mainly due to the nomadic tribes and

villagers of the Rann of Kutch area.

4. The increase in the Khur population in recent times can be attributed to improved agricultural

practices as well as water availability due to the Sardar Sarovar Canal system.

Which of the above statements given above is/are correct?

(a) 1 only (b) 2 and 3 only

(c) 1, 2 and 4 only (d) 1, 2, 3 and 4

Answer: (d)

Explanation

The Indian wild ass also called khur is a subspecies of the onager, native to Southern Asia. As of 2016,

it is listed as Near Threatened by IUCN. It can race at nearly 60 kmph. Native to the Little Rann of

Kutch, the ass is locally known as ‗khur‘. During the early ‘60s and ‘70s it was on the brink of extinction, but has since rebounded, and the head count currently stands at 4,000. The success story

of the Wild Ass is attributed not only to the concerted efforts of the Central and State governments but

also mainly to the nomadic tribes and gutsy villagers of the area. Small and medium farmers growing

groundnut, bajra, cotton, wheat, and rice do not have any skirmishes with the wandering Wild Ass

even if it feeds on their farm produce. Rann is the only place on earth where the endangered Indian

NEO IAS 0484-3190310, 9446331522, 9446334122 9446331522 Page 7 www.neoias.com | www.youtube.com | www.facebook.com/neoias | www.twitter.com/neoias

Wild Ass continues to endure despite all odds. They are usually seen in small herds, especially during the breeding season. The increase in the khur population in recent times can be attributed to

improved agricultural practices as well as water availability due to the Sardar Sarovar Canal system.

The change in cropping patterns, with more cash crops, means that the species receives better forage

and water. This reduces its search for food and thereby helps it conserve its energy in an arid

landscape.

17. With reference to the TRAFFIC, a conservation organisation, consider the following statements:

1. It is a joint programme of WWF and IUCN for wildlife trade monitoring.

2. It is a non-governmental organisation working on trade in wildlife in the context of both

biodiversity conservation and sustainable development. 3. Its global network is research-driven, action-oriented, and committed to delivering innovative and

practical solutions to wildlife trade issues based on the latest information.

Which of the above statements given above is/are correct?

(a) 1 only (b) 2 and 3 only

(c) 1 and 2 only (d) 1, 2 and 3

Answer (d)

Explanation

TRAFFIC, the wildlife trade monitoring network, is a joint program of WWF and IUCN, established by

the WWF & IUCN to respond to the growing threats posed by illegal wildlife trade and overexploitation.

It is an international network, consisting of TRAFFIC International, based in Cambridge, UK. It is the leading non-governmental organization working globally on trade in wild animals and plants in the

context of both biodiversity conservation and sustainable development. Traffic aims to ensure that

trade in wild plants and animals is not a threat to the conservation of nature. TRAFFIC actively

monitors and investigates wildlife trade and provides its information to a diverse audience world-wide,

as a basis for effective conservation policies and programmes. TRAFFIC‘s global network is research-driven, action-oriented, and committed to delivering innovative and practical solutions to wildlife trade

issues based on the latest information. TRAFFIC combines research and analysis to produce guidance

on a wide range of wildlife trade issues, achieving influence through targeted communications, tools

and training.

18. With reference to the helmeted hornbill which was in news recently, consider the following statements:

1. The demand for hornbill ivory has pushed this avian species to the brink. 2. The species is listed as a critically endangered animal by the International Union for Conservation

of Nature‘s (IUCN) red list.

3. It is hunted by the poachers mostly for its brilliant plumage or large bills.

Which of the above statements given above is/are correct?

(a) 1 only (b) 2 and 3 only (c) 1 and 2 only (d) 1, 2 and 3

Answer (c)

Explanation

The helmeted hornbill is being hunted to extinction, one of the latest victims of a thriving global trade in exotic wildlife. The huge birds have been caught for centuries for their tail feathers, prized by local

communities. The Helmeted Hornbill is threatened by poaching and wildlife trafficking, which have

risen significantly owing to increased demand from China for Helmeted Hornbill ivory—the hornbill‘s

unique solid bill casque. Poachers aren’t interested in their brilliant plumage or large bills, but a

helmet-like block of reddish-gold keratin at the front of the skulls known as a casque. By the close of 2015, the species had progressed from vulnerable to critically endangered - leapfrogging two

threat levels to the highest possible risk category on the International Union for Conservation of

Nature‘s ―red list‖.

19. With reference to Bhitarkanika National Park, consider the following statements: 1. The park is home to salt water Crocodile.

2. Bhitarkanika is a protected wetland under Ramsar Convention.

3. The park is included in the UNESCO World Heritage list.

Which of the above statements given above is/are correct?

(a) 1 only (b) 2 and 3 only

(c) 1 and 2 only (d) 1, 2 and 3

Answer: (c)

Explanation

A two-member technical evaluation mission team of International Union for Conservation of Nature (IUCN), deputed by UNESCO, visited the Bhitarkanika National Park. The visit is to perform field

assessment of the unique mangrove ecosystem of the park, which figured on the tentative list of future

NEO IAS 0484-3190310, 9446331522, 9446334122 9446331522 Page 8 www.neoias.com | www.youtube.com | www.facebook.com/neoias | www.twitter.com/neoias

heritage sites of UNESCO in 2009 and made its way to the final list in 2014. The national park presently figures in the listed of protected wetlands under the Ramsar Convention. The Odisha

government had submitted a dossier, compiled by Dehradun-based Wildlife Institute of India,

recommending to UNESCO that the park be declared a World Heritage Site. But it has not been

declared as one so far. Bhitarkanika is a unique ecosystem, highly dynamic and at the same

time fragile. The delta, the river mouth, the sea, mangrove forest, avian fauna, reptiles, amphibians

and fauna and flora contribute to the park‘s biological diversity. In 1974, the Ministry of Forests, in collaboration with UNDP, had started a crocodile hatchery project at Dangmal in the park. The

crocodile population in the park has increased from 96 in 1974 to 1,665 this January.

20. Two important rivers- one with its source in Jabalpur District of Madhya Pradesh, and another, with its source in Bhopal District in Madhya Pradesh – are important tributaries of Yamuna. Interlinking of

these rivers will submerge some parts of an important site of wildlife and biodiversity and a protected

area. Which one of the following could be that Protected area?

(a) Nagarjunasrishailam Tiger reserve

(b) Panna Tiger reserve

(c) Pachmarhi Biosphere resrve (d) Simlipal Tiger reserve

Answer (b)

Explanation

KEN-BETWA PROJECT India‘s first inter State river interlinking project was given a go-ahead by the National Board for

Wildlife (NBWL). The project will bring water to one of India‘s worst drought-affected regions,

Bundelkhand region but in the process will also submerge about 10 per cent of the Panna Tiger

Reserve in Madhya Pradesh, feted as a model tiger-conservation reserve. This would be the first time

that a river project will be located within a tiger reserve. The Government is planned to begin implementing this as a model ILR project. It is insisted on an integrated wildlife management plan.

The main feature of the project is a 230-km long canal and a series of barrages and dams connecting

the Ken and Betwa rivers that will irrigate 3.5 lakh hectares in Madhya Pradesh and 14,000 hectares

of Uttar Pradesh, in Bundelkhand. The key projects are the Makodia dam at Betwa river and

Dhaudhan dam at Ken river, the latter expected to be 77 m high and responsible for submerging 5,803

hectares of tiger habitat in the Panna tiger reserve. There were also concerns that vulture and ghariyal habitat in the region would be affected. The Environment Management Plan (EMP) says only 7.6 per

cent of the park will go under water. Still, some 8,000 hectares of additional space will to be created for

the 37 tigers in residence today. About 6,388 people in 10 villages will be affected due to the

submergence by Daudhan reservoir and 13499 persons living in the 28 villages will be affected due to

the submergence by Makodia reservoir and will have to be resettled. Seventeen lakh residents of nearby towns and villages in both States will benefit from improved drinking water and irrigation

facilities. Chhatarpur, Panna, Tikamgarh, Raisen, and Vidisha districts of Madhya Pradesh and

Mahoba, Jhansi and Banda districts of Uttar Pradesh will benefited from assured irrigation supply,

domestic and industrial water supply and power.

21. With reference to glass-coated ‗manja‘ which was in news recently, consider the following statements:

1. Recently the National Green Tribunal (NGT) imposed an interim nationwide ban on the use of

glass-coated manja.

2. It is banned as the sharp string is likely to pose danger to humans, animals and birds.

Select the correct answer using the code given below. (a) 1 only (b) 2 only

(c) Both 1 and 2 (d) Neither 1 nor 2

Answer (c) Explanation

The National Green Tribunal (NGT) imposed an interim nationwide ban on the use of glass-coated

‗manja‘ for flying kites as the sharp string poses a danger to humans, animals and birds. The ban

order would apply on nylon, Chinese and cotton ‗manja‘ coated with glass. It directed the Manja

Association of India to submit a report to Central Pollution Control Board on harmful effects of kite

strings. The direction came after senior advocate Sanjay Hegde and advocate Shadan Farasat, appearing for People for Ethical Treatment of Animals (PETA), sought the ban, saying Makar Sankranti

was approaching and ‗manja‘ would be used for flying kites. The bench also referred to various orders,

including the November 2015 order of the Allahabad High Court which banned the use of Chinese

‗manja‘ in Uttar Pradesh and sought a ban on the ―manufacture, import, sale and use‖ of the string. In

its petition, PETA contended that the ‗manja‘ posed a grave threat to humans and animals as every year a number of deaths were caused by it. The petition said ‗manja‘ also posed a threat when it came

into contact with live overhead electric wires, leading to grid failure. Due to ‗manja‘ being coated with

NEO IAS 0484-3190310, 9446331522, 9446334122 9446331522 Page 9 www.neoias.com | www.youtube.com | www.facebook.com/neoias | www.twitter.com/neoias

glass, metals and other sharp material, these strings act as good conductors of electricity. PETA averred that children were engaged by the cottage industry to manufacture ‗manja‘ which caused

respiratory problems.

22. A National Park, which lies within one of the global biodiversity hotspots in India, has the world's third

highest peak in it. This park has been recently inscribed as India‘s first Mixed World Heritage Site on

UNESCO World Heritage List. Which of the following could be that National Park? (a) Sunderbans (b) Bhitarkanika

(c) Khangchendzonga (d) Nandadevi

Answer (c) Explanation

Khangchendzonga National Park (KNP), Sikkim has been inscribed as India‘s first “Mixed World

Heritage Site” on UNESCO World Heritage List, by fulfilling the nomination criteria under both

natural and cultural heritage. The KNP exhibits one of the widest altitudinal ranges of any protected

area worldwide. The Park has an extraordinary vertical sweep of over 7 kilometres (1,220m to 8,586m)

within an area of only 178,400 ha and comprises a unique diversity of lowlands, steep-sided valleys and specular snow-clad mountains including the world’s third highest peak, Mt. Khangchendzonga.

Numerous lakes and glaciers, including the 26 km long Zemu Glacier, dot the barren high altitudes.

The KNP lies within the Himalaya global biodiversity hotspot and displays an unsurpassed range

of sub-tropical to alpine ecosystems.

The cultural significance of KNP is portrayed by the multi-layered sacred landscape of Khangchendzonga and the cultural and religious relevance of the hidden land (beyul in Tibetan

Buddhism and Mayel Lyang, in Lepcha tradition) is specific to Sikkim and is a unique example of co-

existence and exchange between different religious traditional and people.

23. Consider the following pairs: Protected areas River flowing

1. Rajaji : Gomati

2. Dandeli Anshi : Kali Ganga

3. Pilihibit : Ganga

Which of the above pairs is/are correctly matched?

(a) 1 and 2 only (b) 3 only (c) 1 and 3 only (d) None

Answer (d)

Explanation 1. Rajaji : Ganga

2. Dandeli Anshi : Kali river

3. Pilihibit : Gomati

Kali ganga (Sarda river – head stream of Ghaghara river) is a river in Uttarakhand. Through Dandeli

Anshi (Kali tiger reserve) the Kali river (Kalinadi) is flowing. The river Gomati is originating from the

Pilihibit.

24. With reference to PETA, which was in news recently, consider the following statements:

1. PETA is a non-governmental organisation dedicated to establishing and protecting the rights of all

animals.

2. PETA achieves its goal through investigative and legislative work, public education, animal rescues,

research, eye-catching demonstrations, celebrity involvement and national media coverage. 3. PETA operates under the principle that animals are not ours to eat, wear, experiment on or use for

entertainment.

Which of the statements given above is/are correct?

(a) 1 only (b) 1 and 3 only

(c) 2 and 3 only (d) 1, 2 and 3

Answer (d)

Explanation

PETA is the largest animal rights organization in the world, with more than 5 million members and

supporters. PETA operates under the simple principle that animals are not ours to eat, wear,

experiment on, or use for entertainment, while educating policymakers and the public about animal abuse and promoting an understanding of the right of all animals to be treated with respect. PETA is

dedicated to establishing and protecting the rights of all animals. PETA works through investigative

and legislative work, public education efforts, research, animal rescues, special events, eye-catching

demonstrations, celebrity involvement, protest campaigns and national media coverage. PETA focuses

its attention on the four areas in which the largest numbers of animals suffer the most intensely for

NEO IAS 0484-3190310, 9446331522, 9446334122 9446331522 Page 10 www.neoias.com | www.youtube.com | www.facebook.com/neoias | www.twitter.com/neoias

the longest periods of time: in the food industry (factory farms), in the clothing trade (leather industries), in laboratories, and in the entertainment industry. It also work on a variety of other

issues, including spaying and neutering, and the killing of ―pests‖, including the cruel killing of

rodents, birds, and other animals who are often considered ―pests‖ as well as cruelty to domesticated

animals.

25. Which of the following areas are identified for the intensive conservation and management of coral

reefs in India?

1. Gulf of Mannar

2. Sundarbans

3. Lakshadweep 4. Gulf of Kutch

5. Andaman and Nicobar

Select the correct answer using the code given below.

(a) 1 and 2 only (b) 1, 3 and 4 only

(c) 1, 3, 4 and 5 only (d) 1, 2, 3, 4 and 5

Answer (c)

Explanation

Coral reefs are the skeletons of stony coral polyps cemented together. It is a diverse underwater

ecosystem held together by calcium carbonate structures secreted by corals. The four major coral reefs

areas identified for intensive conservation and management in India since 1987 are: i) Gulf of Mannar, (ii) Gulf of Kutch; (iii) Lakshadweep; and iv) Andaman and Nicobar Islands. The ministry provides

financial assistance to the state forest departments for all the four identified coral reef areas for

conservation and management of coral and associates. Besides, the ministry also supports R&D

activities with emphasis on targeted research on coral biodiversity, its management and various

aspects of pollution in these areas.

26. Recently, for the first time in our country, which of the following cities has declared the Ganges

dolphin as City Animal?

(a) Allahabad (b) Kolkata

(c) Agra (d) Guwahati

Answer (d)

Explanation

The Ministry of Environment and Forests notified the Ganges River Dolphin as the National Aquatic

Animal. Assam's Guwahati became the first city in the country to have its own city animal with the district administration declaring the Gangetic river dolphin as the mascot. They are listed in

Schedule I of the Wildlife (Protection) Act 1972.

27. With reference to UNESCO‘s Man and the Biosphere (MAB) Programme‘, sometimes seen in the news, consider the following statements:

1. MAB is an Intergovernmental Scientific Programme that aims to establish a scientific basis for the

improvement of relationships between people and their environments.

2. Department of Environment is the nodal agency for Biosphere Reserve programmes in India.

Which of the statements given above is/are correct?

(a) 1 only (b) 2 only (c) Both 1 and 2 (d) Neither 1 nor 2

Answer (c)

Explanation The idea of `Biosphere Reserves‘ was initiated by UNESCO in 1971 under its Man and the Biosphere

(MAB) Programme, as ―Conservation of natural areas and of the genetic material they contain‖.

Launched in 1971, UNESCO‘s Man and the Biosphere Programme (MAB) is an Intergovernmental

Scientific Programme that aims to establish a scientific basis for the improvement of

relationships between people and their environments. MAB combines the natural and social

sciences, economics and education to improve human livelihoods and the equitable sharing of benefits, and to safeguard natural and managed ecosystems, thus promoting innovative approaches to

economic development that are socially and culturally appropriate, and environmentally sustainable.

The Indian National Man and Biosphere Committee constituted by the Central Govt. (Annexure-V)

identify new sites, and it has guided and shaped the programme in India. The National Biosphere

Reserve Programme was initiated in 1986. Biosphere reserves are declared by state or central governments by notification. Once established, the National Governments can nominate them under

the UNESCOs Man & Biosphere (MAB) Programme. If UNESCO accepts the proposal, the biosphere

reserve will enter into World Network of Biosphere Reserves (WNBR) under the MAB Programme.

Department of Environment is nodal agency for Biosphere Reserve programmes.

NEO IAS 0484-3190310, 9446331522, 9446334122 9446331522 Page 11 www.neoias.com | www.youtube.com | www.facebook.com/neoias | www.twitter.com/neoias

28. Consider the following statements:

1. National Parks and Wildlife Sanctuaries are set up with in the legal framework of Wildlife

(Protection) Act, 1972.

2. Tiger Reserves have been given legal protection under Wild Life (Protection) Act, 1972.

Which of the statements given above is/are correct?

(a) 1 only (b) 2 only

(c) Both 1 and 2 (d) Neither 1 nor 2

Answer (c)

Explanation Since a national park is created by central legislation, it has a permanent status. National Parks have

been setup with in the legal framework of Wild Life (protection) Act, 1972. In national park,

human activity is totally prohibited, except the tourism and recreation. Wildlife sanctuary is an

area constituted by a competent authority in which hunting or capturing of animals is prohibited

except by or under control of the highest authority responsible for management of the area. Project Tiger was launched by the Government of India in the year 1973 to save the threatened species of tiger

in the country. Project Tiger is a Centrally Sponsored Scheme of Government of India which was

launched on the 1st of April, 1973 for in-situ conservation of wild tigers in designated tiger reserves.

By the 2006 amendment, the provisions for tiger protection are added to the Wildlife

(Protection) Act 1972.

29. Kuno Sanctuary in Madhya Pradesh was in news related to the

(a) translocation of Asiatic lion from Gir National Park, Gujarat to the sanctuary

(b) translocation of one horned rhino from Kaziranga National Park to the sanctuary

(c) translocation of tigers from Jim Corbett National Park which were affected by the forest fire of Uttarakhand

(d) culling of Nilgai in the sanctuary

Answer (a)

Explanation The translocation of Asiatic lions from Gir National park to Kuno Sanctuary in Madhya Pradesh

was in news recently. Gujarat Government refused to relocate the lions. Experts have for long been

saying that Gir has become overcrowded with lions and there is need to spread them out to other

locations to ensure their genetic stability and health.

30. Consider the following pairs: Species Region

1. Oryx : Uttarakhand

2. Great Indian Bustard : Rajasthan

3. Nilgiri Tahr : Kerala 4. Black Necked Crane : Arunachal Pradesh

Which of the pairs given above is/are correctly matched?

(a) 1 and 2 (b) 1 and 3

(c) 2, 3 and 4 (d) 1, 2 and 4

Answer (c)

Explanation

Also called the Arabian Oryx, the species is adapted to live in hot and arid areas like sandy and stony

deserts. The Arabian Oryx formerly occurred through most of the Arabian Peninsula, north to Kuwait

and Iraq. Now these Sand gazelles are seen at the Arabian Oryx Sanctuary in Umm Al-Zamool, UAE. This species is hunted for sport and food. It is not naturally found in India

31. Consider the following statements:

1. Loktak Lake is a fresh water lake which have Phumdis floating over it.

2. Keibul Lamjao National Park located on the Loktak Lake is a natural refuge of the Sangai (dancing deer, Eld‘s deer).

Which of the statements given above is/are correct?

(a) 1 only (b) 2 only

(c) Both 1 and 2 (d) Neither 1 nor 2

Answer (c)

Explanation

Loktak Lake is the largest fresh water lake in Northeast India. This Lake is known as the only

floating lake in the world due to the floating phumdis (heterogeneous mass of vegetation, soil, and

organic matters at various stages of decomposition) on it. Keibul Lamjao NP in Manipur is the only

NEO IAS 0484-3190310, 9446331522, 9446334122 9446331522 Page 12 www.neoias.com | www.youtube.com | www.facebook.com/neoias | www.twitter.com/neoias

floating park in the world, and is situated on the south bank of loktak lake. The Brow-antlered deer, which was first discovered in Manipur, in the Keibul Lamjao Park area, which necessitated declaring

this reserve park area as a national park to protect and conserve the deer. The species is now confined

to a single small population at the southern end of Loktak Lake in Manipur, India.

32. Consider the following pairs: National Park State

1. Pench

(Indira Priyadarshini) : Maharashtra

2. Sanjay Dubri : Karnataka

3. Sanjay Gandhi (Borivilli) : Madhya Pradesh

Which of the above pairs is/are correctly matched?

(a) 1 only (b) 2 and 3 only

(c) 1 and 3 only (d) None

Answer (d)

Explanation

1. Sanjay Dubri : Madhya Pradesh

2. Sanjay Gandhi (Borivilli) : Maharashtra

3. Pench (Indira Priyadarshini) : Madhya Pradesh

4. Pench (Jawaharlal Nehru) : Maharashtra Sanjay Gandhi (Borivilli) NP in Maharashtra was in news recently, related to the Environment

Ministry‘s notification of defining Eco-sensitive zone around the NP. The Government has reduced the

buffer area of the NP. Pench (Indira Priyadarshini) NP in Madhya Pradesh was in news recently,

because tigers were found dead there due to the poisoning in the Satosha beat area of the said NP.

33. With reference to the Wildlife (Protection) Act 1972, consider the following statements:

1. It was enacted to fulfil India‘s international obligation under the Convention on Biological

Diversity.

2. As per the law only ‗a part of the wild animal‘ is considered as a wildlife trophy.

3. A person can keep the wildlife trophy with the written permission of a Chief Wildlife Warden. Which of the statements given above is/are correct?

(a) 1 only (b) 2 and 3 only

(c) 3 only (d) 1, 2 and 3

Answer (c) Explanation

In 1972, the Wild Life (Protection) Act (WPA) was enacted for the purpose of protection of wild

animals, birds and plants. It was made to prevent hunting and also to control trade in wild life

products. For the purpose of protecting, propagating or developing wildlife and its environment, the

power is conferred on the State Governments and Central Government to proclaim wildlife sanctuaries and national parks. Section 9 of WPA prohibits hunting of any wild animal specified in

Schedule 1, 2, 3 and 4. However the Chief Wild Life Warden may permit hunting of wild animals in

certain cases. The Wildlife (Protection) Act, 1972, has included antler in the definition of wildlife

trophy. A wildlife trophy is defined as the ―whole or any part of any captive animal or wild animal‖.

Section 39 of the Act also states that ―no person shall, without the previous permission in writing

of the Chief Wildlife Warden or the authorised officer acquire or keep in his possession, custody or control or transfer to any person, whether by way of gift, sale or otherwise or destroy or damage such

property.‖ Wildlife and wildlife trophies are considered as owned by the government. The Act also

prescribes imprisonment up to three years and fine of RS. 25,000 for offences involving wildlife

trophies.

34. Consider the following statements regarding ‗National Tiger Conservation Authority (NTCA)‘:

1. NTCA is a statutory body under Ministry of Environment, Forest and Climate Change.

2. NTCA is constituted under Environmental (Protection) Act, 1986.

3. NTCA is chaired by the Prime Minister.

4. NTCA does not intervene in the Project Tiger. Which of the statements given above is/are correct?

(a) 1 only (b) 3 and 4 only

(c) 2 and 3 only (d) 1, 2, 3 and 4

Answer (a)

Explanation

The Govt. of India had launched ―Project Tiger‖ on 1st April 1973 to promote conservation of the

tiger. ‗Project Tiger’ is a Centrally Sponsored Scheme of the Ministry of Environment, Forests

NEO IAS 0484-3190310, 9446331522, 9446334122 9446331522 Page 13 www.neoias.com | www.youtube.com | www.facebook.com/neoias | www.twitter.com/neoias

and Climate Change, providing funding support to tiger range States, for in-situ conservation of

tigers in designated tiger reserves, and has put the endangered tiger on an assured path of

recovery by saving it from extinction. Considering the urgency of the situation, Project Tiger has

been converted into a statutory authority (NTCA) by providing enabling provisions in the Wild Life

(Protection) Act, 1972 through an amendment, viz. Wild Life (Protection) Amendment Act, 2006.

The Wild Life (Protection) Amendment Act, 2006 has come into force with effect from the 4th of

September, 2006, and the NTCA has also been constituted on the same date. National Tiger

Conservation Authority (NTCA) is chaired by the Minister of Environment and Forest. NTCA co-

ordinates, implements and monitors Project Tiger.

35. With reference to ‗Olive Ridley Turtle‘, a reptile found in India, which of the following statements is/are

correct?

1. It is a herbivorous marine animal.

2. It lays eggs on the entire coast of India. 3. It is given legal protection under Schedule I of the Wildlife (Protection) Act, 1972.

Select the correct answer using the code given below.

(a) 1 and 2 only (b) 2 only

(c) 1 and 3 only (d) 3 only

Answer (d)

Explanation

Olive Ridley Turtle OR Olive Back Logger Head Turtle (Lepidochelys olivacea)

The Olive ridley turtles are the smallest and most abundant of all sea turtles found in the world,

inhabiting warm waters of the Pacific, Atlantic and Indian oceans. These turtles, along with their cousin the Kemps ridley turtle, are best known for their unique mass nesting called Arribada, where

thousands of females come together on the same beach to lay eggs. The Olive ridley gets its name from

its olive colored carapace, which is heart-shaped and rounded. They are carnivores, and feed mainly

on jellyfish, shrimp, snails, crabs, molluscs and a variety of fish and their eggs. These turtles spend

their entire lives in the ocean, and migrate thousands of kilometers between feeding and mating

grounds in the course of a year. The coast of Orissa in India is the largest mass nesting site for the Olive-ridley, followed by the coasts of Mexico and Costa Rica. WWF-India, along with the fishermen

community, has been involved in protecting the Olive ridley rookery at the mass nesting site at

Rushikulaya, in Orissa, by fencing off the nesting area and patrolling it till hatching and ensuring a

safe passage for the hatchlings to the sea. Olive-ridleys face serious threats across their migratory

route, habitat and nesting beaches, due to human activities such as turtle unfriendly fishing practices,

development and exploitation of nesting beaches for ports, and tourist centres. Though international trade in these turtles and their products is banned under CITES Appendix I, they are still extensively

poached for their meat, shell and leather, and their eggs, though illegal to harvest, have a significantly

large market around the coastal regions. However, the most severe threat they face is the accidental

killing of adult turtles through entanglement in trawl nets and gill nets due to uncontrolled fishing

during their mating season around nesting beaches. To reduce accidental killing in India, the Orissa government has made it mandatory for trawls to use Turtle Excluder Devices (TEDs), a net specially

designed with an exit cover which allows the turtles to escape while retaining the catch.

36. Vulture population in India perished since 90s possibly due to poisoning after eating carcass of cattle

treated with the anti-inflammatory drug Diclofenac. The drug was banned by the Centre. Another drug which came as an alternative also caused the same effect on the vulture population, it was banned by

the Tamil Nadu Government in 2015. Which one of the following could be that non-steroidal anti-

inflammatory drug (NSAID)?

(a) Aspirin

(b) Ketoprofen (c) Streptomycin

(d) Meloxicam

Answer (b)

Explanation Around 97% of the vulture population in India perished since 90s possibly due to poisoning after

eating carcass of cattle treated with the anti-inflammatory drug Diclofenac. The Centre had banned

veterinary use of the drug in 2006. Vultures act as scavengers, preying on dead animals and

Diclofenac in carcasses led to slow death of vultures. Wildlife activists said that Ketoprofen, which

came as an alternative, caused the same effect on the vulture population. Ketoprofen - a non-steroidal anti-inflammatory drug (NSAID) prescribed by veterinarians as a pain killer for cattle - can kill

vultures when they eat the carcasses of animals treated with it. They cause acute kidney failure in

vultures. Realizing the danger, TN banned the use of the drug in Nilgiris, Coimbatore and Erode in

2015.

NEO IAS 0484-3190310, 9446331522, 9446334122 9446331522 Page 14 www.neoias.com | www.youtube.com | www.facebook.com/neoias | www.twitter.com/neoias

37. With reference to Animal Welfare Board of India (AWBI), consider the following statements:

1. AWBI is established under the Wildlife (Protection) Act, 1972.

2. AWBI is a statutory advisory body on Animal Welfare Laws under the ministry of environment &

forests.

3. It provides financial assistance in the form of grants-in-aid to animal welfare initiatives.

Which of the statements given above is/ are correct? (a) 1 and 3 only (b) 2 only

(c) 2 and 3 only (d) 1, 2 and 3

Answer (c)

Explanation Animal Welfare Board of India (AWBI) was established in 1962 under Section 4 of the Prevention of

Cruelty to Animals Act, 1960 (The parliament of India enacted the PCA Act, 1960 to prevent cruelty

to Animals and to prevent the infliction of unnecessary pain or suffering of animals). AWBI is a

statutory advisory body on Animal Welfare Laws under the ministry of environment & forests is the

country‘s apex institution for promoting welfare of animals with the help of AWBI recognized AWO (Animal Welfare Organisations)/NGO/SPCA(Societies for Prevention of Cruelty to Animals) etc. AWBI is

Government of India‘s largest funding agency for promotion of animal welfare initiatives. AWBI

provides financial assistance in the form of grants-in-aid to Animal Welfare Organisations (AWOs)

registered under the Indian Societies Act, 1890 or Indian trust Act, Co-operative Societies Act,

Gaushala Act, Goseva Ayog Act as charitable non-profit making organisations which have been

recognised by AWBI. The Animal Welfare Board of India (AWBI) scheme relates to provision of assistance for the following type of activities: financial assistance to animal welfare organizations

for maintaining the stray animals in distress and for their treatment (financial assistance based on the

number of animals kept for their fodder, water, minor treatment etc). Human education programmes

for the welfare of animals are implemented by the AWBI. Capital expenditure at the Board‘s

headquarters i.e. expenditure on non-recurring items such as purchase of assets/equipments. Expenditure on a variety of other animal welfare activities such as rescue of cattle from illegal

smuggling and transportation, rehabilitation of rescued circus animals, lab animals, inspections, legal

expenses in connection with court cases pertaining to animal welfare, mobile clinics is also incurred.

38. The ‗United Nations Framework Convention on Biological Diversity (CBD)‘ is an international treaty drawn at

(a) United Nations Conference on the Human Environment, Stockholm, 1972

(b) United Nations Conference on Sustainable Development, Rio de Janeiro, Brazil, 2012

(c) World Summit on Sustainable Development, Johannesburg, 2002

(d) UN Conference on Environment and Development, Rio de Janeiro, 1992

Answer (d)

Explanation

United Nations Conference on Environment and Development (UNCED-Formal name) on

sustainable development have been held in Rio de Janeiro, Brazil in 1992. It was the first global gathering on sustainability. Earth summits main outcome are agenda 21 to promote Sustainable

Development and two important legally binding agreements were opened for signatures: the United

Nations Framework Convention on Climate Change (UNFCCC), to reduce greenhouse gas

emissions; and, the Convention on Biological Diversity (CBD), to conserve biodiversity. UNEP

members made a draft Agreement for the international agreement to protect biodiversity, and they

presented it in the earth summit and the participating countries sign it, and that international agreement is known as Convention on Biological diversity.

39. With reference to ‗National Air Quality Index‘, sometimes seen in the news, consider the following

statements:

1. It is a tool for effective dissemination of air quality information to people by describing as a ‗One

Number- One Colour-One Description‘ 2. Carbon dioxide is a major parameter in the Air quality index.

3. Central Pollution Control Board (CPCB) along with State Pollution Control Boards (SPCB) has been

operating National Air Monitoring Program.

Which of the statements given above is/are correct?

(a) 1 only (b) 2 and 3 only

(c) 1 and 3 only (d) 1, 2 and 3

Answer (c)

Explanation

The new National Air Quality Index (AQI) launched in 2014 can be described as a ‗One Number- One Colour-One Description‘ for the common man to judge the air quality within his vicinity. Air Quality

Index (AQI) is a tool for effective dissemination of air quality information to people. The current

NEO IAS 0484-3190310, 9446331522, 9446334122 9446331522 Page 15 www.neoias.com | www.youtube.com | www.facebook.com/neoias | www.twitter.com/neoias

measurement index for air quality will consider eight pollutants or parameters (PM10, PM2.5, Nitrogen

dioxide (NO2), Sulfur dioxide (SO2), Carbon monoxide (CO), ozone (O3), Ammonia (NH3), and Lead

(Pb)). Central Pollution Control Board (CPCB) along with State Pollution Control Boards (SPCB) has

been operating National Air Monitoring Program (NAMP) covering 240 cities of the country.

40. One of the world heritage site in India, also designated as a Tiger reserve was established as a National

Park specially for conservation and protection of the Greater One Horned Rhinoceros. Which one of the

following could be that National Park?

(a) Sundarbans National Park

(b) Kaziranga National Park

(c) Jim Corbett National Park (d) Rajaji National Park

Answer (b)

Explanation The Kaziranga National Park is one of the oldest wildlife conservancy reserves of India, first notified in

1905 and constituted as Reserved Forest in 1908 with an area of 228.825 Sq. Km specially

established for conservation and protection of the Greater One Horned Rhinoceros (Rhinoceros

Unicornis) whose number was estimated at twenty pairs then. Kaziranga was declared a Game

Sanctuary in 1916 and opened to visitors in 1938. It was declared a Wildlife Sanctuary in 1950, and

notified as Kaziranga National Park in 1974 under the Wildlife (Protection) Act, 1972, with an area of 429.93 Sq. Km. which has now extended to 899 Sq. Km. subsequently. The Kaziranga National park

was declared a Tiger Reserve in the year 2007 and since then it is getting sufficient financial support

under ―CSS Project Tiger‖ which is under the National Tiger Conservation Authority (NTCA). Kaziranga

was also declared as a World heritage site in 1985.

41. With reference to ‗leopards‘, a carnivore animal, which of the following statements is/are correct?

1. Leopards can persist in areas where other large carnivores have been extirpated.

2. It is a species which is extinct in India.

3. It is given legal protection under Schedule I of the Wildlife (Protection) Act, 1972.

Select the correct answer using the code given below. (a) 1 and 2 only (b) 2 only

(c) 1 and 3 only (d) 2 and 3 only

Answer (c)

Explanation Leopards (Panthera pardus) are graceful and powerful big cats closely related to lions, tigers, and

jaguars. The leopard is so strong and comfortable in trees that it often hauls its kills into the branches.

They are highly adaptable and widely distributed; Leopards can persist in areas where other large

carnivores have been extirpated. However, Leopards are declining throughout most of their range. Wild

animal can be deprived of its life and personal liberty only after following due process namely, what is

mentioned in the Wildlife (Protection) Act 1972. Leopard is listed in the Schedule I of the Wildlife (Protection) Act 1972.

42. Consider the following pairs:

Protected Area Well-known for

1. National Chambal

Wildlife Sanctuary,

Madhya Pradesh

:

Gharial

2. Coringa Wildlife Sanctuary, Andhra

Pradesh

: Olive Ridley Turtle

3. Dampa Rain forest,

Mizoram

: Hoolock

Gibbon

Which of the pairs given above is/are correctly matched?

(a) 1 only (b) 1 and 2 only (c) 2 only (d) 1, 2 and 3

Answer (d)

Explanation

The turtles, which are protected under the Wildlife (Protection) Act, commence their journey from the Indian Ocean during their mating season in October and November. When they reach the Bay of

Bengal, the females lay eggs on the beaches. Even as the destination for a majority is Gahirmatha in

Odisha, the sandy stretches of Hope Island of the Coringa Wildlife Sanctuary have turned into a

breeding area for a few hundred turtles every year.

NEO IAS 0484-3190310, 9446331522, 9446334122 9446331522 Page 16 www.neoias.com | www.youtube.com | www.facebook.com/neoias | www.twitter.com/neoias

43. With reference to ‗Kambala‘, sometimes seen in the news, consider the following statements:

1. A traditional sport in which water buffaloes are made to race down a track filled with mud and

water

2. Two water buffaloes, yoked together, were made to run at top speed

3. Kambala used to be held in villages of Tamil Nadu

Select the correct answer using the code given below. (a) 1 and 2 only (b) 3 only

(c) 1 and 3 only (d) 1, 2 and 3

Answer (a)

Explanation After protests in Tamil Nadu managed to achieve a reversal of the ban on bull-taming sport Jallikattu,

many in Karnataka are demanding focus on Kambala -- a traditional sport in which water buffaloes

are made to race down a track filled with mud and water. Under the same order in 2014, the Supreme

Court had banned both Jallikattu and Kambala following complaints by animal welfare groups, which

had pointed to the cruelty and stress inflicted on the animals in both sports. Kambala used to be held in coastal regions of Karnataka. It was especially popular in Udupi and Dakshina Kannada districts. In

Kambala, two water buffaloes, yoked together, were made to run at top speed down a muddy, water-

filled route. They were guided and whipped along by a man who ran with them.

44. Which among the following air pollutants are Criteria air pollutants'? 1. Carbon dioxide.

2. Carbon monoxide

3. Nitrogen dioxide

4. Lead

5. Methane Select the correct answer using the code given below.

(a) 1, 2 and 3 only (b) 2, 3 and 4 only

(c) 1, 4 and 5 only (d) 1, 2, 3, 4 and 5

Answer (b) Explanation

Criteria air pollutants' is a term used internationally to describe air pollutants that have been

regulated and are used as indicators of air quality. The regulations or standards are based on criteria

that relate to health and/or environmental effects. One key feature of criteria air pollutants is that they

are generally widely distributed across the country. The six criteria air pollutants in outdoor (or

ambient) air are: carbon monoxide, lead, nitrogen dioxide, ozone, Particulate Matter and sulfur dioxide.

45. With reference to the Ecosystems, consider the following statements:

1. A biological community interacting with the non-living environment. 2. Set of organisms and abiotic components connected by the exchange of matter and energy.

3. An ecosystem can be visualised as a functional unit of nature.

Which of the statements given above is/are correct?

(a) 1 and 2 only (b) 3 only

(c) 1 and 3 only (d) 1, 2 and 3

Answer (d)

Explanation

The fundamental principle of ecosystem is that at any place where organisms live, there is a

continuous interaction between plants, animals and their environment to produce and exchange materials. OR In other words, an ecosystem can be defined as “The biotic (living) community and its

physical (non-living) environment in which matter cycles and energy flows is called ecosystem.”

The organisms of any community besides interacting among themselves always have functional

relationship with the external world or the environment. This ―structural and functional system of

communities of living organisms and their environment is called ecological system or in short

the ecosystem”

NEO IAS 0484-3190310, 9446331522, 9446334122 9446331522 Page 17 www.neoias.com | www.youtube.com | www.facebook.com/neoias | www.twitter.com/neoias

46. The mortality rate of the lifeline of the desert, ‗Khejri tree‘ is increasing. Why is this a cause of

concern?

1. It has a good role in sustaining the nutrient value of the soil.

2. The amount of Nitrogen in the soil would decrease.

3. The survival of some pollinators like honey bee will be affected.

4. It is a pioneer species in the desert ecosystem. Which of the statements given above is/are correct?

(a) 1, 2 and 3 only (b) 2 only

(c) 1 and 3 only (d) 1, 2, 3 and 4

Answer (a) Explanation

Prosopis cineraria, locally called as Khejri or Jandi is an indigenous tree, which effectively stabilizes

sand dunes and can withstand periodic burial. Culturally, the tree holds a very important place in

lives Rajasthani people, especially the Bishnois.

There is a significant economic and environmental importance too. The Khejri tree plays a vital role in maintaining the ecosystem of the dreary Thar region because of its ability to survive in such

tough conditions and the different ways in which it can be used by the farmers. Apart from being a

source of firewood and fodder, the Khejri also helps in sustaining the nutrient value of the soil and

ensuring a good yield. The khejri tree is adapted to withstand both frost and drought, and

survives both high temperature and low. It is known to improve soil fertility; it aids nitrate

retention in soil. In addition, the fruit is used to make the very popular local dish ‗Sangri’ and can fetch up to Rs 300

per kilo for the farmers. ―Khejri bark is used medicinally for a range of ailments and can be ground up

and made into flour, and famously, saved thousands of lives during the Great Rajputana Famine of

1868. The flowers make an amazing bee-fodder. Even the timber is useful.

The tree contributes to the micro-flora in desert soil. Its tap root goes deep into the earth, to depths of about 25 feet, seeking water. Wells are often dug close to the tree, for one can be sure of finding water

near it. When vultures were more common in the desert, they would roost and nest in its branches.

The decreasing number of Khejri has impacted the culture and economics in some way. In 2015, the

Central Arid Zone Research Institute (CAZRI) took out a report which stated that the number of Khejri

trees per hectare in the 12 dry districts of Rajasthan had dropped to less than 35 per cent. This, the

report stated, was happening because of a very high mortality rate owing to factors like decrease in groundwater level, fungal attacks, indiscriminate cutting, etc. The frequent and almost complete

lopping off of branches for fodder affects seed production and stresses the trees. Frequent drought,

the increased depth of the water table and use of mechanical ploughs - which injure the roots -

have also affected chances of regeneration. One cause of the higher mortality of the khejri tree,

scientists speculate, could also be the changing topography, with the Indira Gandhi Canal making water flow through once arid desert.

47. As a measure to safeguard one of the pristine last bastion of aquatic bio-diversity left relatively

untouched by man, the Ross sea has been declared as the world's largest Marine Protected Area by

(a) National Oceanic and Atmospheric Administration (NOAA) (b) International Whaling Commission

(c) Commission for the Conservation of Antarctic Marine Living Resources (CCAMLR)

(d) International Union for Conservation of Nature (IUCN)

Answer (c) Explanation

At the annual meeting of the United Nations Commission for the Conservation of Antarctic Marine

Living Resources (CCAMLR) to deliberate on the establishment of Marine Protected Areas (MPAs) in

the southern ocean, the pristine last bastion of aquatic bio-diversity left relatively untouched by

man is declared the world's largest Marine Protected Area (MPA) in the Ross Sea in Antarctica. It is the first marine park created in international waters and will set a precedent for further moves

to help the world achieve the International Union for the Conservation of Nature‘s recommendation

that 30% of the world‘s oceans be protected.

National Oceanic and Atmospheric Administration:

NOAA is an agency that enriches life through science. It is an American scientific agency within the

United States Department of Commerce focused on the conditions of the oceans and the atmosphere. Their reach goes from the surface of the sun to the depths of the ocean floor as we work to keep

citizens informed of the changing environment around them. From daily weather forecasts, severe

storm warnings, and climate monitoring to fisheries management, coastal restoration and supporting

marine commerce, NOAA‘s products and services support economic vitality and affect more than one-

third of America‘s gross domestic product. NOAA‘s dedicated scientists use cutting-edge research and

NEO IAS 0484-3190310, 9446331522, 9446334122 9446331522 Page 18 www.neoias.com | www.youtube.com | www.facebook.com/neoias | www.twitter.com/neoias

high-tech instrumentation to provide citizens, planners, emergency managers and other decision

makers with reliable information they need when they need it.

48. With reference to Species richness, consider the following statements:

1. Two communities cannot have the same species richness. 2. Speciation generates new species and adds to species richness.

3. Extinction reduces species richness.

Which of the statements given above is/are correct?

(a) 1 only (b) 2 and 3 only

(c) 1 and 3 only (d) 1, 2 and 3

Answer (b)

Explanation

Species diversity refers to the variety and number of different species in a given time and area.

Ecologically, species variety is measured by species richness and relative abundance of species (species evenness). Species richness refers to the number of species in an area or community.

Generally, species richness or biodiversity in general tends to increase with the size of the area or from

higher to lower latitudes (equator) and higher to lower altitudes. Species richness is the primary

estimation of species in an area. Two communities can have the same species richness, but a

different relative abundance. Speciation generates new species and adds to species richness.

Extinction of species reduces species richness.

49. Consider the following statements:

1. Species diversity refers to the variety and number of different kinds of organisms that make up the

community. 2. A community with an even species abundance (species evenness) is more diverse than one in which

one or two species are abundant.

Which of the statements given above is/are correct?

(a) 1 only (b) 2 only

(c) Both 1 and 2 (d) Neither 1 nor 2

Answer (c)

Explanation

Species diversity refers to the variety and number of different species in a given time and area.

Ecologically, species variety is measured by species richness and relative abundance of species

(species evenness). Species richness refers to the number of species in an area or community. Speciation generates new species and adds to species richness. Extinction of species reduces species

richness. Species evenness means relative abundance of each species in a community. Evenness tells

whether a particular ecosystem is numerically dominated by one species or whether all of its species

have similar abundances. A community with an even species abundance is more diverse than one in

which one or two species are abundant. Two communities can have the same species richness, but a different relative abundance.

50. Consider the following protected areas:

1. Bandipur

2. Eagle Nest 3. Nagarjunsagar Srisailam

4. Rajiv Gandhi Orang

Which of the above are declared Tiger Reserves?

(a) 1 and 2 only (b) 1, 3 and 4 only

(c) 2, 3 and 4 only (d) 1, 2, 3 and 4

Answer (b)

Explanation

Bandipur NP is a Tiger Reserve in Mysore and Chamarajanagar districts of Karnataka.

Eagle Nest wildlife sanctuary is a protected area in West Kameng district of Arunachal Pradesh. Nagarjunsagar Srisailam wildlife sanctuary is a tiger reserve spreads over Guntur, Prakasam &

Kurnool districts of Andhra Pradesh and Nalgonda & Mahaboobnagar districts of Telangana.

Rajiv Gandhi Orang NP is a tiger reserve in Darrang & Sonitpur districts of Assam.

NEO IAS 0484-3190310, 9446331522, 9446334122 9446331522 Page 19 www.neoias.com | www.youtube.com | www.facebook.com/neoias | www.twitter.com/neoias

51. Which of the following pairs is/are correct?

Protected Area Well-known for

1. Kaziranga, Assam :

One horned

rhinoceros

2. Pakke (Pakhui),

Arunachal Pradesh

: Hornbill

3. Rajaji, Uttarakhand : Tiger

4. Mahatma Gandhi

(Wandoor),

Andaman and

Nicobar

: Corals

Select the correct answer using the code given below.

(a) 1 and 2 only (b) 3 and 4 only

(c) 3 only (d) 1, 2, 3 and 4

Answer (d) Explanation

1. Kaziranga, Assam : One horned rhinoceros

2. Pakke (Pakhui), Arunachal Pradesh : Hornbill

3. Rajaji, Uttarakhand : Tiger

4. Mahatma Gandhi (Wandoor), Andaman and Nicobar : Corals

52. Which of the following pairs is/are correct? Biosphere Reserve State

1. Agasthyamalai : Kerala

2. Nanda Devi : Andhra Pradesh 3. Seshachalam : Uttarakhand

4. Khangchendzonga : Sikkim

Select the correct answer using the code given below.

(a) 1 and 2 only (b) 3 and 4 only

(c) 1 only (d) 1 and 4 only

Answer (d)

Explanation

1. Agasthyamalai : Kerala

2. Nanda Devi : Uttarakhand

3. Seshachalam : Andhra Pradesh 4. Khangchendzonga : Sikkim

53. ‗NoMore50‘, sometimes seen in the news, is related to

(a) campaign calling for stronger and higher penalties for Tax evading (b) campaign calling for stronger and higher penalties for animal cruelty

(c) campaign calling for stronger and higher penalties for environmental pollution

(d) campaign calling for stronger and higher penalties for smoking in public place

Answer (b) Explanation

About NoMore50

o The campaign includes a series of videos from celebrities, urging the ministry of environment and

forests to increase punishment for animal cruelty in the Prevention of Cruelty to Animals Act,

1960. o Currently, the maximum penalty even for the most heinous form of animal abuse is a petty Rs 50.

o NoMore50 is a campaign from the animal rights groups - the Humane Society

International/India and the People for Animals.

54. ‗One planet city challenge‘, an initiative to recognize and reward cities for their efforts to combat climate change, is a programme created by

(a) World Wide Fund for Nature (WWF)

(b) International Union for the Conservation of Nature and Natural Resource (IUCN)

(c) United Nations Framework Convention on Climate Change (UNFCCC)

(d) United Nations Conference on Sustainable Development, Rio+20

Answer (a)

Explanation

In cities, everything is closely connected so problems tend to multiply. That‘s the bad news. The good

news is that solutions can also multiply. And solutions exist already today that have the potential of meeting the demands of urban lifestyles, without exhausting the planet‘s ecological capacity.

NEO IAS 0484-3190310, 9446331522, 9446334122 9446331522 Page 20 www.neoias.com | www.youtube.com | www.facebook.com/neoias | www.twitter.com/neoias

So WWF created the One Planet City Challenge to highlight these solutions, and to recognize and reward cities that are busy putting them to use. Cities that aim to provide sustainable housing,

transportation, and energy for their residents while simultaneously acting as inspirational role models

for other cities around the globe.

The One Planet City Challenge, previously known as the Earth Hour City Challenge, invities cities in

participating countries to report ambitious and innovative climate actions, and to demonstrate how

they are delivering on the 2015 Paris Agreement.

55. With reference to ‗sangai‘, a mammal found in India, which of the following statements is/are correct?

1. It is an herbivorous animal.

2. It is found along the entire Western Ghats regions of India. 3. It is given legal protection under Schedule I of the Wildlife (Protection) Act, 1972.

Select the correct answer using the code given below.

(a) 1 and 2 (b) 2 only

(c) 1 and 3 (d) 3 only

Answer (c)

Explanation

About Sangai

o Sangai or Brow- Antlered Deer is an endemic species to Manipur. Once distributed throughout

much of Southeast Asia extending from Manipur in North eastern India to Indochina, it is now

confined in small patches in Manipur (India), Myanmar, Thailand, Cambodia, Lao Viet Nam and Southern China.

o State animal of Manipur

o Its habitat is restricted to the marshy wetland of Keibul Lamjao National Park in the

Southeastern fringe in Loktak Lake (Ramsar Site), Manipur.

o About two-third of the surface area of the lake is occupied by a floating Vegetation which is locally called ‗phumdi‘.

o While walking on the floating biomass, Sangai often balances itself which looks as if it is dancing

on the green grassland therefore popularly called as ‗dancing deer’ of Manipur.

o The Sangai use phumdis, hillocks and elevated strips of land along the lake.

o Found only at Keibul Lamjao National Park, it is the only species from the Northeast, which has

been included in the Centre-sponsored endangered species recovery programme. o It is listed as "endangered" in IUCN Red List and Schedule I of the Indian Wild Life (Protection)

Act, 1972.

o The Wildlife Institute of India has prepared a plan to secure long-term survival of sangai after

extensive research. The Compensatory Afforestation Fund Management and Planning Authority

under the Union ministry of environment, forests and climate change fund the programme.

56. Consider the following pairs:

1. Kamlang Tiger

Reserve

: Arunachal

Pradesh

2. Tadoba-Andhari Tiger Reserve

: Madhya Pradesh

3. Umred-Karhandla

wildlife sanctuary

: Maharashtra

Which of the above pairs is/are correctly matched?

(a) 1 only (b) 2 and 3 only (c) 1 and 3 only (d) 1, 2 and 3

Answer (c)

Explanation

1. Kamlang Tiger Reserve : Arunachal Pradesh 2. Tadoba-Andhari Tiger Reserve : Maharashtra

3. Umred-Karhandla (Umred-Kharngla) wildlife sanctuary : Maharashtra

NEO IAS 0484-3190310, 9446331522, 9446334122 9446331522 Page 21 www.neoias.com | www.youtube.com | www.facebook.com/neoias | www.twitter.com/neoias

57. Consider the following statements:

1. National Board for Wildlife is established under the Environment (Protection) Act, 1986.

2. National Board for Wildlife is a statutory body.

3. National Board for Wildlife is chaired by the Prime Minister.

Which of the statements given above is/are correct?

(a) 1 only (b) 2 and 3 only (c) 2 only (d) 1, 2 and 3

Answer (b)

Explanation

National Board for Wildlife (NBWL) is a statutory Board constituted on 22nd September 2003 under Section 5 of the Wild Life (Protection) Act, 1972. As per the amendment of the Wildlife (Protection)

Act in 2002, a provision was incorporated for the constitution of the National Board for Wildlife,

replacing the Indian Board for Wildlife. The NBWL is chaired by the Hon’ble Prime Minister.

58. If an animal is included in the Vermin category, what does it imply?

(a) The animal comes under the Schedule I of Wildlife (Protection) Act, 1972

(b) That particular animal can be culled as it cause‘s harm to crops and humans

(c) That particular animal is a man-eater wild animal

(d) The animal is given a high degree of protection by Wildlife (Protection) Act, 1972

Answer (b)

Explanation

As per Section 62 of the Wildlife Protection Act, 1972, States can send a list of wild animals to the

Centre requesting it to declare them vermin for selective slaughter. The Central Government may by notification, declare any wild animal other than those specified in Schedule I and part 11 of

Schedule II of the law to be vermin for any area for a given period of time. As long as the

notification is in force such wild animal shall be included in Schedule V of the law, depriving them

of any protection under that law.

Wild boars, nilgai and rhesus monkeys are Shedule II and III members- also protected, but can be

hunted under specific conditions in certain regions of India as it is declared as vermin in those areas. Crows and fruit bat fall in Schedule 5, the vermin category. When a species be declared Vermin under

the Wildlife Act so that they could be culled. The reason for declaring them as vermin is that, the

species may be causing harm to the humans and damaging their agriculture. The main reason for

this problem is their number is allegedly increasing or in another word the carrying capacity is

increased, so there is less resources for the species, hence they are migrating to human fields for their

survival.

59. One of the Ramsar site in India which is unique in being a swamp with floating vegetation that

supports a rich biodiversity, is deteriorating primarily because of the change in water regime due to

construction of a barrages and the commissioning of a Hydro-Power Project in that region. Which of the following could be that Ramsar site?

(a) Bhitarkanika Mangroves

(b) Chilika Lake

(c) East Calcutta Wetlands

(d) Loktak Lake

Answer (d)

Explanation

About Loktak lake

o About two-third of the surface area of the lake is occupied by a floating Vegetation which is locally called ‗phumdi‘.

o A mammal, sangai/dancing deers habitat is restricted to the marshy wetland of Keibul Lamjao

National Park in the Southeastern fringe of in Loktak Lake (Ramsar Site), Manipur.

o The habitat in the Park is deteriorating primarily because of the change in water regime due to

construction of Itahi barrage.

o The phumdi, which used to settle during lean season and get replenished with soil and nourishment, are now continuously floating resulting in their thinning. Consequently, so they are

unable to bear the weight of deer now.

o After the commissioning of the Loktak Hydro-Power Project in 1983, large agricultural areas at

the lake periphery have been submerged which have changed the economic life of the people

making them more dependent on the Park.

NEO IAS 0484-3190310, 9446331522, 9446334122 9446331522 Page 22 www.neoias.com | www.youtube.com | www.facebook.com/neoias | www.twitter.com/neoias

60. Consider the following protected areas:

1. Khangchendzonga

2. Achanakmar Amarkantak

3. Seshachalam

4. Manas

Which of the above are included in the world network of Biosphere reserve by UNESCO? (a) 1 and 3 only (b) 2 only

(c) 1, 2 and 4 only (d) 1, 2, 3 and 4

Answer (b)

Explanation The world network included Biosphere reserves by UNESCO are Nilgiri, Agasthyamalai, Gulf of

Mannar, Great Nicobar, Nanda Devi, Nokrek, Pachmarhi, Achanakmar- Amarkantak, Sunderbans and

Similipal.

61. ‗BIOTA 1.0‘, sometimes seen in the news, is related to

(a) a biofuel

(b) a satellite in PSLV C37

(c) a biodiversity app

(d) an anti-cancer drug produced form plants

Answer (c)

Explanation

o Researchers of the CV Raman Laboratory of Ecological Informatics, which is part of the Indian

Institute of Information Technology and Management – Kerala (IIITM-K), have developed BIOTA, a biodiversity app.

o App is aimed at gathering information on geographical distribution of species for conservation

and educational purposes.

o The app also aims at building up a database on different plant and animal species.

o The beta version of the app, BIOTA 1.0, was released as part of a research paper in the recently-

concluded National Biodiversity Conference in Thiruvananthapuram

62. Consider the following statements regarding the Earth hour:

1. Earth hour is a global environmental movement by International Union for the Conservation of

Nature and Natural Resource (IUCN).

2. Rajkot is declared as the National Earth hour capital of 2016. 3. The global Earth Hour City Challenge (EHCC) is a year-long competition that rewards cities for

their long term efforts to combat climate change.

Which of the statements given above is/are correct?

(a) 1 only (b) 2 and 3 only

(c) 1 and 3 only (d) 1, 2 and 3

Answer (b)

Explanation

Earth Hour is a charitable organisation based out of Singapore. Their mission is uniting people to

protect the planet. WWF is the parent organisation of Earth Hour and started Earth Hour with teams and partners in Sydney, Australia back in 2007. Earth Hour was famously started as a lights-off

event in Sydney, Australia in 2007. Earth hour is a global environmental movement by WWF. Earth

Hour is a people's movement inspiring individuals from more than 7000 cities across the globe to take

a stand against climate change, making it the largest voluntary movement ever witnessed in history.

Even though this is an open source campaign, the Earth Hour brand is legally trademarked worldwide and licensed for use by WWF delivering the Earth Hour movement. Rajkot has been

declared the National Earth Hour Capital 2016 in WWF‘s global Earth Hour City Challenge (EHCC)

for its strong commitments across key areas, backed by strong low carbon actions and community

engagement. The WWF annual EHCC is a year-long competition that highlights and rewards cities

for their substantial long-term efforts to combat climate change. While Paris has emerged as the

Global Earth Hour Capital for this year (2016), Rajkot was also in the run-up to bag the title alongside 17 National Earth Hour Capitals from across the world.

NEO IAS 0484-3190310, 9446331522, 9446334122 9446331522 Page 23 www.neoias.com | www.youtube.com | www.facebook.com/neoias | www.twitter.com/neoias

63. Consider the following statements regarding Eco-sensitive Zone:

1. These are the land falling inside the protected areas which act as the shock absorber to ecological

damages.

2. These are declared to prevent ecological damage caused due to developmental activities around the

protected areas.

3. Eco-sensitive Zones are declared under the Environment (Protection) Act, 1986. Which of the statements given above is/are correct?

(a) 1 only (b) 1 and 3 only

(c) 2 and 3 only (d) 1, 2 and 3

Answer (c) Explanation

Eco-sensitive zones are the transition zones around the Protected areas and is guidelined by the

Ministry of Environment and Forests (MoEF) by the National Wildlife Action Plan (2002-2016). It is

declared as Eco sensitive zone under the Environmental (Protection) Act, 1986. The land falling

within 10km of the boundaries of Protected Areas are declared as Eco sensitive zone/ Eco fragile zone, to prevent ecological damage caused due to developmental activities around National Parks

and Wildlife Sanctuaries by acting as a shock absorber or transition zone. The guidelines for the Eco

sensitive zone were issued in a notification letter dated 19 February, 2011. The methods to manage

such zones are by classifying the activities in to Prohibited, Regulated and Permitted.

64. Which of the following pairs is/are correct? Protected Area State

1. Dibang Wildlife

Sanctuary

: Arunachal

Pradesh

2. Panna Tiger Reserve

: Kerala

3. Wayanad Wildlife Sanctuary

: Madhya Pradesh

4. Dandeli - Anshi

Tiger Reserve

: Karnataka

Select the correct answer using the code given below.

(a) 1 and 2 (b) 3 and 4

(c) 1 only (d) 1 and 4

Answer (d)

Explanation

1. Dibang Wildlife Sanctuary : Arunachal Pradesh

2. Panna Tiger Reserve : Madhya Pradesh

3. Wayanad Wildlife Sanctuary : Kerala

4. Dandeli - Anshi Tiger Reserve : Karnataka

65. With reference to ‗hangul‘, a mammal found in India, which of the following statements is/are correct?

1. It is a subspecies of elk native to India.

2. It is found along the entire deccan regions of India

3. It is given legal protection under Schedule I of the Wildlife (Protection) Act, 1972. Select the correct answer using the code given below.

(a) 1 and 2 (b) 2 only

(c) 1 and 3 (d) 3 only

Answer (c)

Explanation

About Kashmir Stag

o The Kashmir Stag or Hangul is a subspecies of elk native to India.

o Known for its giant antlers bearing 11 to 16 points.

o The main concentration of the endangered species is in Srinagar‘s famed Dachigam national park. o Once found in the high altitudes of northern India and Pakistan, the animal now only lives in the

dense riverine forests of Dachigam.

Protection Measures o Listed under the Schedule-I of the Wildlife (Protection) Act, 1972 and J&K Wildlife (Protection)

Act, 1978.

o Listed among the top 15 species of high conservation priority by the Central Government.

NEO IAS 0484-3190310, 9446331522, 9446334122 9446331522 Page 24 www.neoias.com | www.youtube.com | www.facebook.com/neoias | www.twitter.com/neoias

66. With reference to an organisation known as ‗Wetlands International‘, which of the following statements

is/are correct?

1. It is a global organisation dedicated to maintaining and restoring wetlands for their environmental

values as well as for the services they provide to people.

2. It identifies the sites known as ‗Important Bird and Biodiversity Areas‘.

3. They work at the field level to develop and mobilise knowledge, giving respect to traditional values and use the practical experience to advocate for better policies.

Select the correct answer using the code given below.

(a) 1 only (b) 2 and 3 only

(c) 1 and 3 only (d) 1, 2 and 3

Answer (c)

Explanation

Wetlands International (WI)

o WI is a global not-for-profit organisation established in 1995, headquartered in the Netherlands

o Wetlands International is dedicated to maintaining and restoring wetlands— for their environmental values as well as for the services they provide to people

o They are also supported by government and NGO membership.

o They work at the field level to develop and mobilise knowledge, giving respect to traditional

values and use the practical experience to advocate for better policies

67. With reference to coral bleaching, consider the following statements:

1. Coral bleaching takes place when the symbiotic relationship between algae (zooxanthellae) and

their host corals breaks down under certain environmental stresses

2. Coral beaching occurs only due to the rise in temperature

Which of the statements given above is/are correct? (a) 1 only (b) 2 only

(c) Both 1 and 2 (d) Neither 1 nor 2

Answer (a)

Explanation Coral bleaching

Coral bleaching takes place when the symbiotic relationship between algae (zooxanthellae) and

their host corals breaks down under certain environmental stresses. This results in the host

expelling their zooxanthellae. In the absence of symbiotic algae, the corals expose their white

underlying calcium carbonate coral skeleton and the affected coral colony becomes pale in colour.

Coral bleaching can be activated and persist during varied environmental stresses When corals are stressed by changes in conditions such as temperature, light, or nutrients, they expel

the symbiotic algae living in their tissues, causing them to turn completely white.

Coral reefs are very sensitive ecosystems – even slight changes in water temperature and salinity can

lead to coral ―bleaching‖ and even death.

68. With reference to the International Union for Conservation of Nature and Natural Resources (IUCN),

which of the following statements is/are correct?

1. IUCN is a membership union composed of both government and civil society organisations (NGOs)

for the conservation of biodiversity

2. IUCN has an official Observer Status at the United Nations General Assembly 3. IUCN runs field projects around the world to better manage natural environments.

Select the correct answer using the code given below.

(a) 1 only (b) 2 and 3 only

(c) 1 and 3 only (d) 1, 2 and 3

Answer (d)

Explanation

International Union for Conservation of Nature (IUCN)

o IUCN is also known as the World Conservation Union

o Former name - International Union for the Protection of Nature (IUPN) o Membership union composed of both government and civil society organisations (NGOs) for

the conservation of biodiversity

o Established in 1948, headquarters in Gland, Switzerland

o Has official Observer Status at the United Nations General Assembly

o Prepare a list called IUCN Red List (Red Data Book), which classify the plants and animals on the basis of their threat

o It provides public, private and non-governmental organisations with the knowledge and tools that

enable human progress, economic development and nature conservation to take place

together.

NEO IAS 0484-3190310, 9446331522, 9446334122 9446331522 Page 25 www.neoias.com | www.youtube.com | www.facebook.com/neoias | www.twitter.com/neoias

o It is a leading provider of conservation data, assessments and analysis. o They run hundreds of field projects around the world to better manage natural environments.

69. Which one of the following is the best description of World Heritage Sites?

(a) A place of importance that is listed by the United Nations Educational, Scientific and Cultural

Organization (UNESCO) as of special cultural or physical significance. (b) The sites of outstanding example of ongoing ecological process and beauty only.

(c) The sites protected by IUCN without any human interventions.

(d) The places having ecologically important wetlands which are protected by Ramsar Convention.

Answer (a) Explanation

World Heritage Sites

o A place of importance that is listed by the United Nations Educational, Scientific and Cultural

Organization (UNESCO) as of special cultural or physical significance.

o UNESCO seeks to encourage the identification, protection and preservation of cultural, and natural heritage around the world considered to be of outstanding value to humanity. This is embodied in

an international treaty called the World Heritage Convention concerning the Protection of the

World Cultural and Natural Heritage, adopted by UNESCO in 1972.

o There are natural and cultural sites.

For Natural WHS, the site must be of outstanding universal value, the site should be of outstanding example ongoing ecological process, it should contain outstanding unique

natural phenomena and beauty, contain exceptional biodiversity.

For the Cultural WHS it should represent a masterpiece of human creative, monumental arts, outstanding architecture, traditional settlement their cultural tradition or civilisation,

etc

o There are 7 Natural, 27 Cultural and 1 Mixed World Heritage Sites in India.

70. With reference to an initiative called Asian Waterbird Census (AWC), which of the following statements is/are correct?

1. The AWC is a citizen science programme that supports the management and conservation of

waterbirds and wetlands.

2. AWC is only monitoring waterbirds.

3. The Bombay Natural History Society (BNHS) along with Wetlands International is organizing Asian Waterbird Census (AWC) to count waterbirds all over India.

Select the correct answer using the code given below.

(a) 1 only (b) 2 and 3 only

(c) 1 and 3 only (d) 1, 2 and 3

Answer (c)

Explanation

About AWC

o The Asian Waterbird Census (AWC), conducted each year in January, is a waterbird and

wetland-monitoring programme initiated in 1987. o AWC is a part of the International Waterbird Census (IWC) which is completing its 50 years of

waterbird monitoring.

o The Bombay Natural History Society (BNHS) along with Wetlands International is organizing

Asian Waterbird Census (AWC) to count waterbirds all over India.

Objectives of AWC

o The AWC is a citizen science programme that supports the management and conservation of

waterbirds and wetlands.

o The data collected through the AWC would also help identify and protect new sites of

importance for waterbirds.

o Obtain information of waterbird population on an annual basis o Annual monitoring of the status and condition of wetlands

o Obtain information of waterbird population on an annual basis and use it as a basis for monitoring

population

o The AWC encourages people to count waterbirds in wetlands around them and collect information

that would help promote the designation and management of internationally important sites such as nationally protected areas, Ramsar sites and Important Bird and Biodiversity Areas (IBAs).

NEO IAS 0484-3190310, 9446331522, 9446334122 9446331522 Page 26 www.neoias.com | www.youtube.com | www.facebook.com/neoias | www.twitter.com/neoias

71. With reference to India Biodiversity Awards (IBA), which of the following statements is/are correct?

1. It is a joint initiative of the Ministry of Environment, Forest and Climate Change (MoEFCC),

National Biodiversity Authority (NBA) and United Nations Development programme (UNDP).

2. It recognise the contribution of stakeholders towards the conservation of biodiversity and

excellence in biodiversity governance.

3. It is only giving awards to the steps taken for conservation of threatened species. Select the correct answer using the code given below.

(a) 1 only (b) 1 and 2 only

(c) 2 and 3 only (d) 1, 2 and 3

Answer (b) Explanation

o India Biodiversity Awards is a joint initiative of the Ministry of Environment, Forest and Climate

Change (MoEFCC), National Biodiversity Authority (NBA) and United Nations Development

programme (UNDP).

o It Recognise the contribution of stakeholders towards the conservation of biodiversity and excellence in biodiversity governance.

o The first India Biodiversity Awards were jointly announced by the Ministry and UNDP India in 2012

at the high level segment of the Eleventh meeting of Conference of Parties (COPs) to the

Convention on Biological Diversity, during India‘s Presidency of the COPs to the Convention.

o In 2016, the categories of the India Biodiversity Awards have been modified to link them with the

provisions of the Biological Diversity Act, 2002, in order to incentivize stakeholders for the conservation of biodiversity and the effective implementation of the Act.

The four categories of awards:

1. Conservation of threatened species

2. Sustainable use of biological resources 3. Successful mechanisms/models for access and benefit sharing

4. Biodiversity Management Committee (BMC)

72. What would happen if excess fertilisers are run off in to a lake ecosystem?

1. The excess in nutrients results in algal bloom 2. Biodiversity of the lake will increase

3. Decrease in the oxygen level of the lake and becomes dead zones

Which of the statements given above is/are correct?

(a) 1 only (b) 1 and 3 only

(c) 2 and 3 only (d) 1, 2 and 3

Answer (b)

Explanation

The application of Nitrogen fertilizers to crops has caused increased rates of denitrification and

leaching of nitrate into groundwater. The additional Nitrogen entering the groundwater system eventually flows into streams, rivers, lakes, and estuaries. In these systems, the added Nitrogen can

lead to eutrophication. Increased deposition of Nitrogen from atmospheric sources is also because of

fossil fuel combustion and forest burning. Both of these processes release variety forms of Nitrogen

through combustion. Livestock release a large amount of ammonia into the environment from their

wastes. These all Nitrogen enters the soil system and then the hydrologic system through leaching,

groundwater flow, and runoff. Eutrophication is the natural aging of a lake by nutrient enrichment of its water. Pollutants from

man‘s activities like effluents from the industries and homes can radically accelerate the aging process.

This phenomenon has been called Cultural or Accelerated Eutrophication. During the past century,

lakes in many parts of the earth have been severely eutrophied by sewage and agricultural and

industrial wastes. The prime contaminants are nitrates and phosphates, which act as plant nutrients. They overstimulate the growth of algae, causing unsightly scum and unpleasant odors,

and robbing the water of dissolved oxygen vital to other aquatic life. At the same time, other pollutants

flowing into a lake may poison whole populations of fish, whose decomposing remains further

deplete the water‘s dissolved oxygen content. In such fashion, a lake can literally choke to death.

Excessive quantities of chemicals which get washed from the fields act as nutrients for algae in the

nearby water body to flourish (algal bloom - excessive growth of planktonic (free-floating) algae). Algal blooms cause deterioration of the water quality and fish mortality. Once these algae die, they serve as

food for decomposers like bacteria. A lot of oxygen in the water body gets used up. This results in a

decrease in the oxygen level which may kill aquatic organisms, and becomes dead zones. Some bloom-

forming algae are extremely toxic to human beings and animals.

NEO IAS 0484-3190310, 9446331522, 9446334122 9446331522 Page 27 www.neoias.com | www.youtube.com | www.facebook.com/neoias | www.twitter.com/neoias

73. A snow covered region is the natural habitat of an Indian species, which were not sighted in the

Kargil‘s Drass Sector after the 1999 Kargil war, due to its habitat destruction. The species is again

spotted in that region recently. Which one of the following could be that animal?

(a) Himalayan Musk Deer

(b) Himalayan Brown Bear

(c) Himalayan Tahr (d) Snow leopard

Answer (b)

Explanation

About Himalayan Brown Bear o It‘s an omnivorous animal and India‘s largest animal (mammal) in the Himalayas.

o They exhibit sexual dimorphism.

o It is distributed in Nepal, Pakistan, and northern India.

o In India they are found in Jammu and Kashmir, Himachal Pradesh and Uttarakhand.

o The best place to spot them is the Great Himalayan National Park in Himachal. o A brown bear requires about 100 square kilometer as its territory to survive and any human

intervention disturbs its ecology.

Threatened by

o They are poached for their fur and claws for ornamental purposes and internal organs for use in

medicines.

o They are killed by shepherds to protect their livestock. o The tree bearing the state flower of Himachal — buransh — is the favourite hangout of this bear.

Due to the high value of the buransh tree, it is being commercially cut causing further destruction

to the brown bear’s home.

o In Himachal, their home is the Kugti and Tundah wildlife sanctuaries and the tribal Chamba

region. o The 1999 Kargil war had caused significant destruction to their habitats, and no Himalayan bear

had been spotted in the region for many years. The Tiger Hill, one of the battle fields, was a

prominent brown bear habitat. The war and the movement of troops did impact the animal‘s

behavior and ecology.

74. With reference to speciation, consider the following statements:

1. Speciation occurring when the population becomes separated by geographical barriers is called as

allopatric speciation.

2. The formation of two or more species from a single ancestral species all occupying the same

geographical area is known as sympatric speciation. Select the correct answer using the codes given below.

(a) 1 only (b) 2 only

(c) Both 1 and 2 (d) Neither 1 nor 2

Answer (c) Explanation

The speciation occurring when the population becomes separated by geographical barriers is called

allopatric speciation. When a population is geographically continuous, the allele frequencies among

its members are similar; however, when a population becomes separated, the allele frequencies

between the two groups can begin to vary. Thus the chance of interbreeding between these populations

is reduced. If the separation between groups continues for a long period of time, the differences between their alleles can become more and more pronounced due to differences in climate, predation,

food sources, and other factors, eventually leading to the formation of a new species. Each separated

population acquires mutations by natural selection to adapt to the new environment. A long time,

reproductive isolation sets in separating two populations into two species. Allopatric speciation events

can occur either by dispersal, when a few members of a species move to a new geographical area, or by vicariance, when a natural situation, such as the formation of a river or valley, physically divide

organisms. Allopatric is the most common form of speciation.

Sympatric Speciation: It is the formation of two or more species from a single ancestral species all occupying the same geographical area. In this the populations are not geographically separated.

Sympatric speciation often occurs through polyploidy. A diploid individual cannot interbreed with

tetraploid individual leading to reproductive isolation. This type of speciation is rare and occurs often

in plants as self fertilization and polyploidy is common in plants compared to animals.

NEO IAS 0484-3190310, 9446331522, 9446334122 9446331522 Page 28 www.neoias.com | www.youtube.com | www.facebook.com/neoias | www.twitter.com/neoias

75. With reference to ‗pacha chedi‘ (Neurocalyx calycinus), sometimes seen in the news, consider the

following statements:

1. It is a medicinal plant endemic to the southern parts of Western Ghats and Sri Lanka.

2. It has a potential for developing anti-cancer drug.

Which of the statements given above is/are correct?

(a) 1 only (b) 2 only (c) Both 1 and 2 (d) Neither 1 nor 2

Answer (c)

Explanation

Neurocalyx calycinus is a medicinal plant endemic to the southern parts of Western Ghats and Sri Lanka. It could offer scientists the key to new herbal formulations and modern drugs for the treatment

of cancer and wounds and burns. Scientists at the Jawaharlal Nehru Tropical Botanic Garden and

Research Institute (JNTBGRI) here have confirmed the multiple therapeutic properties of Neurocalyx

calycinus used by the Cholanaickan tribe, one of the particularly vulnerable groups in Kerala, to treat

inflammations and wounds. The researchers have filed for a patent on a novel herbal drug formulation

possessing wound-healing, burn-healing, anti-cancer, analgesic, anti-inflammatory, immuno- enhancing, platelet-augmentation and anti-oxidant effects. The presence of high Vitamin E content

and potent cytoprotective activity in cell lines in the plant species have also enhanced the prospects of

developing an anti-cancer drug.

76. With reference to National Green Corps (NGC), consider the following statements:

1. It is a national programme conceptualized and initiated by the Ministry of Environment and

Forests, Government of India.

2. It aims at building cadres of young children working towards environmental conservation and

sustainable development.

3. It is operated through Eco-clubs set up in schools registered as members of NGC. Which of the statements given above is/ are correct?

(a) 1 and 3 only (b) 2 only

(c) 2 and 3 only (d) 1, 2 and 3

Answer (d)

Explanation

National Green Corps (NGC) is a major initiative of MOEFCC for creating environmental awareness

launched in 2001-02 which aims at building cadres of young children working towards environmental

conservation and sustainable development. The phenomenal response that NGC has received and has

made the network more than 1,00,000 Eco clubs across the country in 14 years, making it one of the largest conservation networks indicates its importance at grass root level in taking the environment

awareness at mass.

The unique partnership between the MoEF, the state Government agencies along with the dedicated

NGOs, working in the field of Environmental Education has contributed to the success of the

programme. It is operated through Eco-clubs set up in schools registered as members of NGC, this programme exposes school children to in-depth field experiences, and provides opportunities to

convert their ideas into creative action.

The programme has a cascading effect, seeks to redirect the consciousness of students towards

environment friendly attitudes and actions and goes beyond schools, promoting school-society

interactions to sensitize the society. Also in order to strengthen monitoring mechanism of NGC

programme, MoEFCC is in process to establish Management Information system (MIS) which will open up vistas in sharing and accessing the information on NGC among all stakeholders. The MIS reporting

and monitoring would be interactive and creative in ensuring NGC mobility and services.

77. Consider the following statements:

1. Marine National Park in Gulf of Kutch on the Jamnagar coast of Gujarat is the first National Marine Park of India.

2. Mahatma Gandhi Marine National Park is located in Wandoor near Port Blair on the south western

coast of south Andaman.

Which of the statements given above is/are correct?

(a) 1 only (b) 2 only (c) Both 1 and 2 (d) Neither 1 nor 2

Answer (c)

Explanation

India‘s first Marine Wildlife Sanctuary and first Marine National Park were created in the Gulf of Kutch (Jamnagar- Gujarat) in 1980 and 1982, respectively. It is an archipelago of 42 tropical islands along

the northern coast of Jamnagar district and the southern coast of Kutch.

NEO IAS 0484-3190310, 9446331522, 9446334122 9446331522 Page 29 www.neoias.com | www.youtube.com | www.facebook.com/neoias | www.twitter.com/neoias

Mahatma Gandhi Marine National Park is located in Wandoor near Port Blair on the south western coast of south Andaman of Andaman and Nicobar Islands (Union territory) of India. The Park was

established in the year of 1983 for the protection of marine life including corals and nesting sea

turtles. The Park has dense mangrove vegetation and sand covered beaches with a tropical type of

climate. The landscape of the Park is dotted with 15 rocky islands covered with tropical forest.

78. The gas released mostly from landfills in urban areas is

(a) Oxygen (b) Hydrogen

(c) Nitrogen (d) Methane

Answer (d) Explanation

Landfill gas (LFG) is a natural by product of the decomposition of organic material in landfills. LFG is

composed of roughly 50 percent methane (the primary component of natural gas), 50 percent carbon

dioxide (CO2) and a small amount of non-methane organic compounds. Methane is a potent

greenhouse gas 28 to 36 times more effective than CO2 at trapping heat in the atmosphere over a 100-year period, as per the latest Intergovernmental Panel on Climate Change (IPCC) assessment report

(AR5).

Landfill gases are produced when bacteria break down organic waste. The amount of these gases

depends on the type of waste present in the landfill, the age of the landfill, oxygen content, the amount

of moisture, and temperature. For example, gas production will increase if the temperature or moisture

content increases. Though production of these gases generally reaches a peak in five to seven years, a landfill can continue to produce gases for more than 50 years.

79. Consider the following statements:

1. Spruce and Cedar are tree varieties of coniferous forest. 2. Sal is a part of Himalayan vegetation.

3. Khejri tree is a dominant tree in the north eastern rain forest.

Which of the statements given above is/are correct?

(a) 1 only (b) 2 and 3 only

(c) 1 and 3 only (d) 1, 2 and 3

Answer (a)

Explanation

Khejri tree or Jandi tree plays a vital role in preserving the ecosystem of arid and semi-arid areas.

And it is a species in the arid regions. Basically jandi tree is a symbol of socio-economic

development of the arid regions. Since all the parts of the tree are useful, it is called kalp taru. It is also known as the ‗king of desert‘, and the ‗wonder tree‘. Sal tree is a deciduous tree and is a part

of deciduous forests.

80. With reference to The Scheduled Tribes and Other Traditional Forest Dwellers (Recognition of Forest Rights) Act, 2006, consider the following statements:

1. The Act provides for the restitution of deprived forest rights, including individual rights to

cultivated land in forestland and community rights over common property resources.

2. The onus of implementation of the Forest conservation Act lies with the Central Government.

3. The Act recognise forest rights of forest dwelling Scheduled Tribes (FDSTs) who have been

occupying the land before October 25, 1980. 4. The Ministry of Environment, Forest and Climate Change is the nodal agency for implementing the

provisions of the Act.

Which of the statements given above is/are correct?

(a) 1 only (b) 2, 3 and 4 only

(c) 1 and 3 only (d) 1, 2, 3 and 4

Answer (c)

Explanation

The Act seeks to recognize and vest rights for habitation and occupation in forest land for forest

dwelling Scheduled Tribes (STs)as well as Other traditional forest dwellers (OTFDs) who have been residing in such forests for generations but whose rights could not be recorded. The Act is an enabling

legislation with a motive to undo the historical injustice done to these communities. The Act was

notified for operation with effect from 31.12.2007.

The act seeks to recognize and vest certain forest rights in the forest dwelling Scheduled Tribes and

other traditional forest dwellers. The act provides for the restitution of deprived forest rights across India, including both individual rights to cultivated land in forestland and community rights over

common property resources.

NEO IAS 0484-3190310, 9446331522, 9446334122 9446331522 Page 30 www.neoias.com | www.youtube.com | www.facebook.com/neoias | www.twitter.com/neoias

The Scheduled Tribes (Recognition of Forest Rights) Act, 2006 recognise forest rights of forest dwelling Scheduled Tribes (FDSTs) who have been occupying the land before October 25, 1980. The Act

extends to the whole of India except the State of Jammu and Kashmir.

The onus of implementation of the Forest conservation Act lies with the State/UT Governments. The

Ministry of Tribal Affairs is the nodal agency for implementing the provisions of the Scheduled

Tribes and Other Traditional Forest Dwellers (Recognition of Forest Rights) Act, 2006 popularly known

as FRA, 2006.

81. With reference to Red Panda, consider the following statements:

1. It is a mammal found in Eastern Himalayas.

2. It survives by grazing only on bamboo forest. 3. It is given legal protection under Schedule I of the Wildlife (Protection) Act, 1972.

Which of the statements given above is/are correct?

(a) 1 only (b) 2 and 3 only

(c) 1 and 3 only (d) 1, 2 and 3

Answer (c)

Explanation RED PANDA (Ailurus fulgens)(EN)

It is primarily a herbivore. The name panda is said to have come from the Nepali word ‗ponya,‘ which

means bamboo or plant eating animal. In India it is found in only three states: Sikkim, West Bengal

and Arunachal Pradesh. It can also be seen in the Nokrek of Meghalaya, but studies on the species say that it is a subspecies of Red Panda. Almost 50 percent of the red panda’s habitat is in the

Eastern Himalayas. Red Panda is closely associated with montane forests (oak mixed; mixed broad-

leaf conifer; and conifer) with dense bamboo-thicket understorey. Red Panda prefers to live near

(typically within 100–200 m of) water. It is largely arboreal. Red Panda is largely vegetarian, eating

chiefly young leaves and shoots of bamboo. It also takes fruit, roots, succulent grasses, acorns,

lichens, birds' eggs and insects. The loss of nesting trees and bamboo is causing a decline in red panda populations across much of their range because their forest home is being cleared. The major

threats are habitat loss and fragmentation; habitat degradation; and physical threats and also

climate change and natural disasters. They are poached for their distinctive pelts. Red panda fur caps

or hats have been found for sale in Bhutan. It is included in Appendix I of CITES. It is listed in

Schedule I of the Indian Wild Life (Protection) Act 1972.

82. Consider the following pairs:

Wetlands State

1. Chilika Lake : Orissa

2. Keoladeo Ghana National Park

: Manipur

3. Kolleru Lake : Andhra Pradesh

4. Wular Lake : Punjab

Which of the above pairs is/are correctly matched?

(a) 1, 2 and 3 only (b) 1 and 3 only (c) 2 and 4 only (d) 1, 2, 3 and 4

Answer (b)

Explanation

Chilika Lake : Orissa Keoladeo Ghana National Park : Rajasthan

Kolleru Lake : Andhra Pradesh

Wular Lake : Jammu & Kashmir

83. With reference to the transpiration, which of the following statements is/are correct? 1. Transpiration is the evaporation of water into the atmosphere from the leaves.

2. The rate of transpiration increases with decreasing humidity.

3. The rate of transpiration decreases with decreasing light intensity.

Select the correct answer using the code given below.

(a) 1 only (b) 2 and 3 only (c) 1 and 3 only (d) 1, 2 and 3

Answer (d)

Explanation

Transpiration is the evaporation of water into the atmosphere from the leaves. The rate of transpiration increases with decreasing humidity. The rate of transpiration decreases with decreasing light intensity.

NEO IAS 0484-3190310, 9446331522, 9446334122 9446331522 Page 31 www.neoias.com | www.youtube.com | www.facebook.com/neoias | www.twitter.com/neoias

84. Which one of the following term describes the process of using microbes to treat areas of land or sea

that have been contaminated by pesticides, oil or solvents?

(a) Ammonification

(b) Bioremediation

(c) Eutrophication

(d) Nitrification

Answer (b)

Explanation

Bioremediation: "Remediate" means to solve a problem, and "bio-remediate" means to use biological

organisms to solve an environmental problem such as contaminated soil or groundwater. In a non-polluted environment, bacteria, fungi, protists, and other microorganisms are constantly at work

breaking down organic matter. Bioremediation works by providing these pollution-eating organisms

with fertilizer, oxygen, and other conditions that encourage their rapid growth. These organisms would

then be able to break down the organic pollutant at a correspondingly faster rate. In fact,

bioremediation is often used to help clean up oil spills.

Bioremediation provides a good cleanup strategy for some types of pollution, but it will not work for

all. For example, bioremediation may not provide a feasible strategy at sites with high concentrations

of chemicals that are toxic to most microorganisms. These chemicals include metals such as cadmium

or lead, and salts such as sodium chloride.

Nonetheless, bioremediation provides a technique for cleaning up pollution by enhancing the same biodegradation processes that occur in nature. Depending on the site and its contaminants,

bioremediation may be safer and less expensive than alternative solutions such as incineration or

landfilling of the contaminated materials. It also has the advantage of treating the contamination in

place so that large quantities of soil, sediment or water do not have to be dug up or pumped out of the

ground for treatment.

85. India is a signatory to Ramsar Convention. Ramsar Convention is related to conservation and wise use

of

(a) Water birds

(b) Desert Vegetation (c) Forest lands

(d) Wetlands

Answer (d)

Explanation The importance and usefulness of wetlands was first brought to notice of the world through a

Convention on Wetlands held at the Iranian city Ramsar, in the year 1971 February 3rd. The

Convention on Wetlands, called the Ramsar Convention, is an intergovernmental treaty with 135

contracting parties that provides the framework for national action and international cooperation

for the conservation and wise use of wetlands and their resources. The Convention defines wise use of wetlands as ―the maintenance of their ecological character, achieved through the implementation

of ecosystem approaches, within the context of sustainable development‖. Wise use can thus be

seen as the conservation and sustainable use of wetlands and all the services they provide, for the

benefit of people and nature.

World Wetlands Day is celebrated every year on 2 February, the date in 1971 that the Convention on

Wetlands was adopted in the Iranian city Ramsar on the shores of the Capsian Sea. The World Wetland Day marks the date of the signing of the Convention on Wetlands.

86. With reference to the cactus, a plant lives in hot and dry habitats, which of the following statements

is/are correct? 1. They uses Crassulacean acid metabolism in photosynthesis process.

2. Their leaves are reduced to spines to reduce water loss.

3. The stem of cactus does the photosynthesis.

Select the correct answer using the code given below.

(a) 1 only (b) 2 and 3 only

(c) 1 and 3 only (d) 1, 2 and 3

Answer (d)

Explanation

Cactus is a type of plant that can store large amounts of water and survive in extremely hot and dry habitats. Almost all cacti are native to deserts and dry regions of South and North America. Due

to their attractive morphology, cacti can be found throughout the world today. Over collecting and

habitat loss are major threats to the survival of cacti in the wild. Certain species are listed as

endangered, but luckily, trade of most species of cacti is prohibited by law. They have waxy substance

NEO IAS 0484-3190310, 9446331522, 9446334122 9446331522 Page 32 www.neoias.com | www.youtube.com | www.facebook.com/neoias | www.twitter.com/neoias

on the surface which prevents loss of water via transpiration. Cacti have spines instead of leaves. Spines can be soft or rigid, straight or curved, arranged in rows or scattered. They can reach 6 inches

in length. Spines have two major roles: they prevent loss of water via transpiration and keep the

plant safe from animals. Since cacti live in dry areas, they need to absorb large amount of water and

store it in the stem and roots for the periods of drought. Besides storing of water, stem plays role in

the process of photosynthesis (production of food by using the sunlight and carbon dioxide). Water

from cactus has higher density compared with tap water, but it is safe for drinking. Cacti can survive for 15 to 300 years, depending on the species.

In general, plants photosynthesize through their leaves and, to a much lesser extent, through their

stems. While different cactuses may have specialized photosynthetic techniques, in general,

photosynthesis occurs in a cactus' stem or trunk. The concept of photosynthesis in a cactus or

succulent plant may seem strange considering that most of them do not have obvious leaves like many other plants, but their stems or trunks serve the same photosynthetic function as some other plants'

leaves.

In plants photosynthesis takes place in chloroplasts. Chloroplasts may be in the cells of fruits, stems,

but most of all in leaves. Cacti perform photosynthesis in the same place as all other plants, inside

cellular organelles called chloroplasts. Stem succulents have thick water storing stems. To limit

water loss many desert adapted plants have very small or vestigial leaves. This presents a trade- off problem because reducing water loss with smaller leaves also reduces the number of chloroplast

containing cells in the leaves. To compensate, the photosynthesis function is transferred to cortical

cells in the fleshy stem, and so their green colour. This is not so unusual because many plants have

green stems when they are young. Photosynthetic stems must also have stomate, pores, through which

gas exchange takes place. Again there is a functional trade-off problem because while gas exchange is necessary for photosynthesis, water is lost through the pores. Cacti are among a group of plants that

solve this problem by capturing solar energy by day, and finishing the photosynthesis by night

when the stomata can be open with minimal water loss. They use Crassulacean acid metabolism

(CAM). The stomata are open at night, and CO2 is fixed by the enzyme PEP carboxylase (and some

others) into malic acid, which is stored in the vacuoles of cells in the cortex and pith. During the

daytime, with closed stomata, CO2 is split from the malic acid and used in the Calvin cycle like other plants.

87. Consider the following pairs:

Protected areas well known for

1. Manas : One Horned Rhinoceros

2. Ranthambhore : Tiger

3. Periyar : Elephant

Which of the statements given above is/are correct?

(a) 1 only (b) 2 and 3 only (c) 1 and 3 only (d) 1, 2 and 3

Answer (d)

Explanation

Protected areas Well known for 1. Manas (Assam) : One Horned Rhinoceros

2. Ranthambhore (Rajasthan) : Tiger

3. Periyar (Kerala) : Elephant

88. With reference to the natural vegetation of India, consider the following statements:

1. Deodar is a coniferous tree found in Himalaya.

2. Shola forests are found mainly on the upper reaches of Himalaya.

3. Teak and Sal are found in the moist deciduous forest.

4. 90% of the total area of mangrove forest in India is mainly concentrated in Gujarat.

Which of the statements given above is/are correct? (a) 1 only (b) 2, 3 and 4 only

(c) 1 and 3 only (d) 1, 2, 3 and 4

Answer (c)

Explanation Coniferous trees like pine (chir pine), deodar, silver fir, Spruce etc are part of Himalaya. Coniferous

trees are mostly associated with cold regions. Shola forests are the patches of stunted tropical

montane forest found in valleys within rolling grassland in the montane regions of South India.

Teak trees are the most dominant species of trees found in tropical deciduous forests. Bamboos,

sal, shisham, sandalwood, khair, kusum, arjun, mulberry are some of the other commercially

NEO IAS 0484-3190310, 9446331522, 9446334122 9446331522 Page 33 www.neoias.com | www.youtube.com | www.facebook.com/neoias | www.twitter.com/neoias

important species found here. The largest area of mangrove forest in the world is in Sundarbans in

the west Bengal.

89. With reference to Bombay Natural History Society (BNHS), which of the following statements is/are

correct?

1. It is a Governmental organisation in India for wildlife research.

2. They are trying to conserve the nature through scientific based research, education and Public

awareness.

3. It is the Partner of BirdLife International in India. Which of the statements given above is/are correct?

(a) 1 only (b) 2 and 3 only

(c) 2 only (d) 1, 2 and 3

Answer (b)

Explanation

BNHS-India, is a pan-India wildlife research organization, has been promoting the cause of nature

conservation for the past 132 years, since 1883. It is an oldest and one of the largest environmental

non-governmental organisations in India. BNHS Mission is Conservation of Nature, primarily

Biological Diversity through action based on Research, Education and Public Awareness. BNHS Vision is to become a Premier independent scientific organization with a broad based

constituency, excelling in the conservation of threatened species and habitats. This society is engaged

in collecting information and specimens of fauna and flora all through the country and played an

important role in drawing public attention to the need of wildlife conservation. During its journey

from an amateurs‘ organization to a professional organization, the guiding principle of BNHS has been that conservation should be based on scientific research. Over 150 scientists and

NEO IAS 0484-3190310, 9446331522, 9446334122 9446331522 Page 34 www.neoias.com | www.youtube.com | www.facebook.com/neoias | www.twitter.com/neoias

professionals work on and off the field to further the tasks of research, conservation and nature

education. BNHS has been designated as a Scientific and Industrial Research Organization (SIRO)

by Department of Science & Technology, Government of India and is the Partner of BirdLife

International in India. In the true spirit of sustainable development, BNHS aims to continue its

conservation mission with a professional approach and strong research background, while closely

working with various stakeholders including government, research institutes, academia, corporates, civil society groups and the general public.

90. With reference to the Global Tiger Forum (GTF), which of the following statements is/are correct?

1. It is an international non-governmental organization.

2. They are providing leadership throughout the world in order to safeguard the survival of the Tiger, its prey and the habitat.

3. GTF is giving membership for members from the Tiger range countries only.

Which of the statements given above is/are correct?

(a) 1 only (b) 2 and 3 only

(c) 2 only (d) 1, 2 and 3

Answer (c)

Explanation

The Global Tiger Forum was established in 1994, with a goal to highlight the rationale for Tiger

preservation and provide leadership and common approach throughout the world in order to safeguard the survival of the Tiger, its prey and the habitat. The Global Tiger Forum is an inter-

governmental international organization for the conservation of tigers in the wild. The GTF is

committed to work through mandate ratified by the Tiger Range Countries (TRCs) to ensure the

implementation of the Global Tiger Recovery Programme (GTRP). It is the only inter-governmental

and international body functioning exclusively for the conservation of tigers in the wild. GTF is involved in various activities and programs in the tiger range countries. The GTF has several members

from the Tiger range countries, non-Tiger range countries, international and national NGOs.

The tiger range countries are Bangladesh, Bhutan, Cambodia, China, India, Indonesia, Lao PDR,

Malaysia, Myanmar, Nepal, Russia, Thailand and Vietnam and the Non Tiger Range countries willing

to support and/or participate in Tiger conservation programmes is United Kingdom.

In the decision of the International Symposium on Tiger held at New Delhi during February 1993 wherein a Delhi declaration on tiger conservation was adopted by the participants of the conference,

there was a recommendation that a Global Tiger Forum be set up to embark on a worldwide campaign

to save the wild tiger. This was followed up in the meeting of the Forestry forum for developing

countries held at New Delhi in Sept '93 where a small sub group meeting was taken by the Minister in

charge of Environment and Forests, Government of India with the Ministers of various tiger range states present. This sub group also re-iterated the need for formation of Global Tiger Forum. As a

follow up of the decision, the first meeting of the tiger range countries was held in March 1994 at

New Delhi attended by 11 tiger range states and others. In the said meeting a resolution was passed

to form the Global Tiger Forum with its secretariat at New Delhi and the Union Minister for

Environment & Forest, Govt. of India as the first Chairman of this forum.

The current Chairperson of the forum is the Minister of Environment and Forests, Government of India, who took over the responsibilities of GTF after the 5th General Assembly held at New Delhi,

India on March 30, 2011.

91. With reference to Convention on the Conservation of Migratory Species of Wild Animals (CMS),

sometimes seen in news, consider the following statements: 1. It is an environmental treaty under the aegis of IUCN.

2. CMS provides a global platform for the conservation and sustainable use of migratory animals and

their habitats.

3. The convention also deals with the migration routes of the migratory species.

Which of the statements given above is/are correct? (a) 1 and 2 only (b) 2 and 3 only

(c) 1 and 3 only (d) 1, 2 and 3

Answer (b)

Explanation CONVENTION ON THE CONSERVATION OF MIGRATORY SPECIES OF WILD ANIMALS (CMS) OR

BONN CONVENTION

As an environmental treaty under the aegis of the United Nations Environment Programme

(UNEP), CMS provides a global platform for the conservation and sustainable use of migratory

animals and their habitats. CMS brings together the States through which migratory animals pass, the Range States, and lays the legal foundation for internationally coordinated conservation measures

throughout a migratory range. As the only global convention specializing in the conservation of

migratory species, their habitats and migration routes, CMS complements and co-operates with a

NEO IAS 0484-3190310, 9446331522, 9446334122 9446331522 Page 35 www.neoias.com | www.youtube.com | www.facebook.com/neoias | www.twitter.com/neoias

number of other international organizations, NGOs and partners in the media as well as in the corporate sector.

Migratory species threatened with extinction are listed on Appendix I of the Convention. CMS

Parties/states strive towards strictly protecting these animals, conserving or restoring the places where

they live, mitigating obstacles to migration and controlling other factors that might endanger them.

Migratory species that need or would significantly benefit from international co-operation are listed in

Appendix II of the Convention. The Agreements may range from legally binding treaties to less formal instruments, such as Memoranda of Understanding, and can be adapted to the requirements of

particular regions. The development of models tailored according to the conservation needs throughout

the migratory range is a unique capacity to CMS. Adopted in Bonn, Germany, on 23 June 1979, it

entered into force on 1 November 1983. India is a party of CMS since 1983.

The Government of India signed the ‗Raptor MoU‘, covering 76 species, out of which 46 including vultures, falcons, eagles, owls, hawks, kites, harriers and others are also found in India. The

agreement was signed on March 7, 2016 at the Convention on Migratory Species (CMS) in Abu Dhabi.

The agreement, under Article IV paragraph 4 of the CMS, is not “legally binding”.

92. With reference to Nagoya Protocol, sometimes seen in news, consider the following statements: 1. It is a supplementary agreement to the Convention on Biological Diversity.

2. It refers to the Convention of International Trade in Endangered Species of Wild Fauna and Flora.

3. It provides a transparent legal framework for the fair and equitable sharing of benefits arising out

of the utilization of genetic resources.

4. India is not a signatory to the Protocol. Which of the statements given above is/are correct?

(a) 1, 2 and 3 only (b) 1 and 3 only

(c) 2 and 4 only (d) 1, 2, 3 and 4

Answer (b) Explanation

Nagoya Protocol on Access and Benefit Sharing (ABS)

The Nagoya Protocol on Access to Genetic Resources and the Fair and Equitable Sharing of

Benefits Arising from their Utilization (ABS) to the Convention on Biological Diversity is a

supplementary agreement to the Convention on Biological Diversity. It provides a transparent legal

framework for the effective implementation of one of the three objectives of the CBD: the fair and equitable sharing of benefits arising out of the utilization of genetic resources. The Nagoya

Protocol on ABS was adopted on 29 October 2010 in Nagoya, Japan and entered into force on 12

October 2014, 90 days after the deposit of the fiftieth instrument of ratification. Its objective is the fair

and equitable sharing of benefits arising from the utilization of genetic resources, thereby contributing

to the conservation and sustainable use of biodiversity. India signed the Protocol on 11th May 2011, and ratified it on 9th October, 2012.

Convention of International Trade in Endangered Species of Wild Fauna and Flora (CITES)

CITES is an international agreement between governments which aims to ensure that

international trade in specimens of wild animals and plants does not threaten their survival. The

purpose of CITES is to ensure that wild fauna and flora in international trade are not exploited

unsustainably. CITES is an international convention that combines wildlife and trade themes with a legally binding instrument for achieving conservation and sustainable use objectives. The Convention

establishes an international legal framework together with common procedural mechanisms for the

strictest control of international commercial trade in species threatened with extinction, and for an

effective regulation of international trade in others.

93. Consider the following:

1. Manas Wildlife Sanctuary

2. Mukundpur Sanctuary

3. Bhandhavgarh National Park

Which of the above is/are World Heritage Sites as per UNESCO? (a) 1 only (b) 1 and 2 only

(c) 1, 2 and 3 (d) None

Answer (a)

Explanation A UNESCO World Heritage Site is a place of importance that is listed by the United Nations

Educational, Scientific and Cultural Organization as of special cultural or physical significance. The

United Nations Educational, Scientific and Cultural Organization (UNESCO) seeks to encourage the

identification, protection and preservation of cultural, and natural heritage around the world

NEO IAS 0484-3190310, 9446331522, 9446334122 9446331522 Page 36 www.neoias.com | www.youtube.com | www.facebook.com/neoias | www.twitter.com/neoias

considered to be of outstanding value to humanity. This is embodied in an international treaty called the World Heritage Convention concerning the Protection of the World Cultural and Natural Heritage,

adopted by UNESCO in 1972. There are natural and cultural sites. For Natural WHS, the site must be

of outstanding universal value, the site should be of outstanding example ongoing ecological process,

it should contain outstanding unique natural phenomena and beauty, contain exceptional

biodiversity. For the Cultural WHS it should represent a masterpiece of human creative,

monumental arts, outstanding architecture, traditional settlement their cultural tradition or civilisation, etc

Great Himalayan National Park Conservation Area, Western Ghats, Nanda Devi and Valley of

Flowers National Parks, Sundarbans National Park, Kaziranga National Park, Keoladeo National

Park and Manas Wildlife Sanctuary are the natural world heritage sites in India.

Mukundpur (Madhya Pradesh), the 25-hectare sanctuary at Mukundpur, satna district, 20km from Rewa is the World‘s first sanctuary for white tiger.

94. With reference to mangrove trees, consider the following statements:

1. Mangroves are salt tolerant trees.

2. Mangrove forest in India can be found along the coastal areas of West Bengal and Odisha only. 3. Mangrove forests only grow at tropical and subtropical latitudes near the equator.

Which of the statements given above is/are correct?

(a) 1 and 2 (b) 2 only

(c) 1 and 3 only (d) 3 only

Answer (c)

Explanation

Mangroves are salt-tolerant forest ecosystems of tropical and sub-tropical regions of the world. They

have a special type of trees called the Mangrove vegetation (a group of trees and shrubs). Mangrove

forests only grow at tropical and subtropical latitudes near the equator because they cannot withstand freezing temperatures. The mangrove ecosystems constitute a symbiotic link or bridge

between terrestrial and marine ecosystems. They are found in the inter-tidal zones of sheltered shores,

estuaries, creeks, backwaters, lagoons, marshes and mud-flats.

Mangroves anchor themselves to the soil by sending out long roots from trunks and branches. These

act as stilts to support the tree and prevent it from toppling over. The partly submerged roots of

mangrove trees spread out beneath the water to trap sediment and prevent it being washed out to sea, thereby stabilize bottom sediments. The muddy water where they live contains very little oxygen,

but mangroves have an ingenious solution to this problem too. The trees send out a second air-

breathing root system.

The mangrove forest can be found in West Bengal, Orissa, Tamil Nadu, Gujarat and Andhra

Pradesh.

95. Government of India encourages the cultivation of ‗Jatropha‘. What is the importance of this plant?

1. It is a rich source of biodiesel and produces high protein manure.

2. It is well adapted to thrive under a wide range of physiographic and climatic conditions.

3. It is pest resistant, easily propagated, has a low gestation period and has a high seed yield and oil content.

4. Grazing of the plant by cattle or sheep is the main problem facing for the cultivation of the plant.

Which of the statements given above is/are correct?

(a) 1 only (b) 3 and 4 only

(c) 1, 2 and 3 only (d) 1, 2, 3 and 4

Answer (c)

Explanation

Biodiesel in India is mostly produced from the oils extracted from the seeds of Jatropha, mainly

because of the fact that edible oil is scarce and the country already depends on huge quantity of imported oils for edible purposes. Apart from Jatropha, Pongamiapinnata, Mahua, Neem and Castor

are also considered as good source of non-edible oil-based biodiesel in India. In Western countries,

biodiesel is typically made from vegetable oil (rapeseed oil, sunflower oil and palm oil), animal tallow

and used cooking oil. Rapeseed oil has 82 percent of the share of the world‘s biodiesel feedstock

followed by sunflower oil, soybean and palm oil.

Jatropha is a genus of nearly 175 species of shrubs, low-growing plants, and trees. The discussions of Jatropha as a biodiesel plant actually means a particular species of the plant, Jatropha curcas. The

plant is indigenous to parts of Central America, however it has spread to other tropical and

subtropical regions in Africa and Asia. Jatropha curcas is a perennial shrub that, on average, grows

approximately three to five meters in height.

By virtue of being a member of the Euphorbiacea family, Jatropha has a high adaptability for thriving under a wide range of physiographic and climatic conditions. It is found to grow in almost all parts of

NEO IAS 0484-3190310, 9446331522, 9446334122 9446331522 Page 37 www.neoias.com | www.youtube.com | www.facebook.com/neoias | www.twitter.com/neoias

the country up to an elevation 3000 feet. Jatropha is a perennial plant, suitable for all soils

including degraded and barren lands. It occupies limited space hence is highly suitable for

intercropping. Extensive research has shown that jatropha requires low water and fertilizer for

cultivation, is not grazed by cattle or sheep, is pest resistant, is easily propagated, has a low

gestation period and has a high seed yield and oil content. It also produces high protein manure.

96. Consider the following pairs:

Protected Area Well-known for

1. Jaldapara National

Park

: One horned

Rhinos

2. Namdapha National Park

: Lion

3. Gir National Park : Flying Squirrel

Which of the pairs given above is/are correctly matched?

(a) 1 only (b) 1 and 2 only

(c) 2 only (d) 1, 2 and 3

Answer (a)

Explanation

Protected Area Well-known for

1. Namdapha National Park : Flying Squirrel 2. Jaldapara National Park : One horned Rhinos

3. Gir National Park : Lion

97. Consider the following pairs: 1. Nagarjunsagar

Srisailam Tiger

Reserve

: Krishna river

2. Agasthyamalai

Biosphere Reserve

: Neyyar river

3. Keibul Lamjao National Park

: Loktak lake

Which of the above pairs is/are correctly matched?

(a) 1 only (b) 2 and 3 only

(c) 1 and 3 only (d) 1, 2 and 3

Answer (d)

Explanation

Nagarjunsagar Srisailam Tiger Reserve : Krishna

Agasthyamalai Biosphere Reserve : Neyyar

Keibul Lamjao National Park : Loktak lake Keibul Lamjao National Park (KLNP) is situated inside the Loktak lake.

98. Consider the following:

1. Star tortoise

2. Puma 3. Palm civet

4. Sangai deer

Which of the above are naturally found in India?

(a) 1 and 3 only (b) 1, 3 and 4 only (c) 2 and 4 only (d) 1, 2, 3 and 4

Answer (b)

Explanation

The Puma is a large, secretive Cat predominantly found in the mountains from southern Canada to

the tip of South America. Puma is not found naturally in India. Star tortoise, Palm civet and

Sangai deer are found naturally in India.

NEO IAS 0484-3190310, 9446331522, 9446334122 9446331522 Page 38 www.neoias.com | www.youtube.com | www.facebook.com/neoias | www.twitter.com/neoias

99. With reference to Seshachalam Biosphere Reserve, sometimes seen in news, consider the following

statements:

1. It is located in the Western Ghats

2. It is having unique tropical rain forest

3. Red sanders is a dominant tree variety found in the Reserve

Which of the statements given above is/are correct? (a) 1 and 2 only (b) 2 only

(c) 2 and 3 only (d) 3 only

Answer (d)

Explanation

It is located in the hill ranges of Eastern Ghats in southern Andhra Pradesh. It has group of seven

hills. It includes Sri Venkateswara NP and WLS. It is home to a number of endemic species including

the famous Red Sanders and Slender Loris.

100. Consider the following pairs:

1. Hastinapur Wildlife : Uttar Pradesh

Sanctuary

2. Kalesar NP : Haryana 3. Daphla hills : Arunachal Pradesh

Which of the above pairs is/are correctly matched?

(a) 1 only (b) 2 and 3 only

(c) 1 and 3 only (d) 1, 2 and 3

Answer (d)

Explanation

All are Correct

1. Hastinapur Wildlife : Uttar Pradesh

Sanctuary 2. Kalesar NP : Haryana

3. Daphla hills : Arunachal Pradesh

NEO IAS 0484-3190310, 9446331522, 9446334122 9446331522 Page 39 www.neoias.com | www.youtube.com | www.facebook.com/neoias | www.twitter.com/neoias